Как решить линейное уравнение с одной переменной 7 класс: Линейное уравнение с одной переменной. Алгоритм решения. Алгебра, 7 класс: уроки, тесты, задания.

Содержание

Линейное уравнение с одной переменной. Алгоритм решения. Алгебра, 7 класс: уроки, тесты, задания.




















1.

Корень линейного уравнения


Сложность:
лёгкое

1


2.

Решение линейного уравнения


Сложность:
лёгкое

1


3.

Линейное уравнение, схема решения


Сложность:
лёгкое

1


4.

Линейное уравнение (коэффициент при x дробный)


Сложность:
лёгкое

1


5.

Составление и решение линейного уравнения


Сложность:
лёгкое

2


6.

Линейное уравнение вида x + a = b


Сложность:
лёгкое

1


7.

Линейное уравнение вида x + a = 0


Сложность:
лёгкое

1


8.

Линейное уравнение вида ax + b = 0


Сложность:
лёгкое

1


9.

Линейное уравнение (с дробями)


Сложность:
среднее

2


10.

Линейное уравнение вида a — kx = c


Сложность:
среднее

3


11.

Линейное уравнение вида a — b + kx = c + d — mx


Сложность:
среднее

4


12.

Задача на движение


Сложность:
среднее

3


13.

Задача на движение, скорость по течению и против течения


Сложность:
среднее

4


14.

Задача на движение, две лодки


Сложность:
среднее

4


15.

Задача на движение в одном направлении


Сложность:
среднее

4


16.

Задача на движение, скорость течения реки


Сложность:
сложное

5


17.

Решение уравнения, записанного в виде пропорции


Сложность:
сложное

3


18.

Определение книг на полках


Сложность:
сложное

6

Линейное уравнение с одной переменной с примерами.

п.1. Количество корней линейного уравнения с одной переменной

Линейным уравнением с одной переменной x называют уравнение вида ax = b, где a и b — действительные числа.
a называют коэффициентом при переменной , а b — свободным членом .

При решении линейных уравнений возможны три случая.

a

b

x

Количество корней

$b \in \Bbb R$ — любой

$x = \frac{b}{a}$

$x \in \Bbb R$ — любой

Бесконечное множество корней

$x \in \Bbb \varnothing $

п.2. Примеры

Пример 1. Решите уравнение 6-5x = 8(3,5-2x)

Решение:

$ 6-5x = 8(3,5-2x) \iff 6-5x = 28-16x \iff -5x+16x = 28-6 \iff $

$ \iff 11x = 22 \iff x = 2 $

Ответ: x=2

Пример 2. Решите уравнение $\frac{2}{3} x-\frac{4}{5} = 0,6x$

Решение:

$ \frac{2}{3}x-\frac{4}{5} = 0,6x | ×15 \iff 2x∙5-4∙3 = 0,6x∙15 \iff 10x-12=9x \iff $

$ \iff 10x-9x = 12 \iff x = 12 $

Ответ: x = 12

Пример 3. 2-3a)}{a} = \frac{a(a-3)}{a} = a-3 \end{array} \right.} \\ {\left\{ \begin{array}{c} a = 0 \\ 0x = 0 \end{array} \right.} \end{array} \right. \iff \left[ \begin{array}{cc} {\left\{ \begin{array}{c} a≠0 \\ x = a-3 \end{array} \right.} \\ {\left\{ \begin{array}{c} a = 0 \\ x \in \Bbb R \end{array} \right.} \end{array} \right. $$

Ответ: при a ≠ 0,x = a-3; при a = 0, $x \in \Bbb R$ — любой

Пример 6*. Решите уравнение (k+1)x = k

Решение:

$$ (k+1)x = k \iff \left[ \begin{array}{cc} {\left\{ \begin{array}{c} k+1 ≠ 0 \\ x = \frac{k}{k+1} \end{array} \right.} \\ {\left\{ \begin{array}{c} k+1 = 0 \\ 0x = -1 \end{array} \right.} \end{array} \right. \iff \left[ \begin{array}{cc} {\left\{ \begin{array}{c} k ≠ -1 \\ x = \frac{k}{k+1} \end{array} \right.} \\ {\left\{ \begin{array}{c} k = -1 \\ x \in \Bbb \varnothing — решений \quad нет \end{array} \right.} \end{array} \right. $$

Ответ: при k ≠ -1, $ x = \frac{k}{k+1} $, при k = -1 решений нет

Пример 7*. Решите уравнение ax+b = cx+d

Решение:

$$ ax+b = cx+d \iff ax-cx = d-b \iff (a-c)x = d-b \iff $$

$$ \left[ \begin{array}{cc} {\left\{ \begin{array}{c} a-c ≠ 0 \\ x = \frac{d-b}{a-c} \end{array} \right.} \\ {\left\{ \begin{array}{c} a-c = 0 \\ d-b = 0 \\ 0x = 0 \end{array} \right.} \\ {\left\{ \begin{array}{c} a-c = 0 \\ d-b ≠ 0 \\ 0x ≠ 0 \end{array} \right.} \end{array} \right. \iff \left[ \begin{array}{cc} {\left\{ \begin{array}{c} a ≠ c \\ x = \frac{d-b}{a-c} \end{array} \right.} \\ {\left\{ \begin{array}{c} a = c \\ d = b \\ x \in \Bbb R — любой \end{array} \right.} \\ {\left\{ \begin{array}{c} a = c \\ d ≠ b \\ x \in \Bbb \varnothing — решений \quad нет \end{array} \right.} \end{array} \right. $$

Макарычев 7. Уравнения с одной переменной

Ознакомительная версия с цитатами из учебника для принятия решения о покупке. Алгебра. 7 класс. Учебник / Макарычев Ю.Н., Миндюк Н.Г., Нешков К.И. и др.; под ред. С.А. Теляковского — 8-е издание. М.: Просвещение (2018). ГЛАВА I учебника. § 3. УРАВНЕНИЯ С ОДНОЙ ПЕРЕМЕННОЙ.  (6. Уравнение и его корни. 7. Линейное уравнение с одной переменной. 8. Решение задач с помощью уравнений. Упражнения №№ 111 — 166. Контрольные вопросы и задания. Дополнительные упражнения №№ 233 — 252 к параграфу 3)

§ 3. УРАВНЕНИЯ С ОДНОЙ ПЕРЕМЕННОЙ

6. Уравнение и его корни

Рассмотрим задачу: «На нижней полке в 4 раза больше книг, чем на верхней. Если с нижней полки переставить на верхнюю 15 книг, то книг на полках станет поровну. Сколько книг на верхней полке?»

Обозначим буквой х число книг на верхней полке. Тогда число книг на нижней полке равно 4х. Если с нижней полки переставить на верхнюю 15 книг, то на нижней полке останется 4х – 15 книг, а на верхней будет х + 15 книг. По условию задачи после такой перестановки книг на полках окажется поровну. Значит, 4х – 15 = х + 15.

Чтобы найти неизвестное число книг, мы составили равенство, содержащее переменную. Такие равенства называют уравнениями с одной переменной или уравнениями с одним неизвестным.

Нам надо найти число, при подстановке которого вместо х в уравнение 4х – 15 = х + 15 получается верное равенство. Такое число называют решением уравнения или корнем уравнения.

Из уравнения 4х – 15 = х + 15 находим, что 4х – х = 15 + 15,   3х = 30,   х = 10.

Уравнение 4х – 15 = х + 15 имеет один корень – число 10.

Можно привести примеры уравнений, которые имеют два, три и более корней или не имеют корней.

Так, уравнение (х – 4)(х – 5)(х – 6) = 0 имеет три корня: 4, 5 и 6. Действительно, каждое из этих чисел обращает в нуль один из множителей произведения (х – 4) (х – 5) (х – 6), а значит, и само произведение. При любом другом значении х ни один из множителей в нуль не обращается, а значит, не обращается в нуль и произведение. Уравнение х + 2 = х не имеет корней, так как при любом значении х левая часть уравнения на 2 больше его правой части.

Уравнение х2 = 4 имеет два корня – числа 2 и –2. Уравнение (х – 2)(х + 2) = 0 также имеет корни 2 и –2. Уравнения, имеющие одни и те же корни, называют равносильными уравнениями. Уравнения, не имеющие корней, также считают равносильными.

При решении уравнений используются следующие свойства:

Например, равносильны уравнения 5х = 2х + 7 и 5х – 2х = 7, равносильны также уравнения 6х = 2х + 8 и 3х = х + 4.

Указанные свойства уравнений можно доказать, опираясь на свойства числовых равенств: если к обеим частям верного равенства прибавить одно и то же число или обе части верного равенства умножить или разделить на одно и то же отличное от нуля число, то получится верное равенство.

Упражнения

7. Линейное уравнение с одной переменной

Каждое из уравнений 5х = –4, –0,2x = 0, –х = –6,5 имеет вид ах = b, где х – переменная, а и b – числа. В первом уравнении а = 5, b = –4, во втором а = –0,2, b = 0, в третьем а = –1, b = –6,5.

Такие уравнения называют линейными уравнениями с одной переменной.

Выясним, сколько корней может иметь линейное уравнение. Рассмотрим уравнение ах = b, в котором коэффициент а не равен нулю. Разделив обе части уравнения на а, получим х= b/a. Значит, линейное уравнение ах = b, в котором а ≠ 0, имеет единственный корень b/a.

Рассмотрим уравнение ах = b, у которого коэффициент а равен нулю. Если а = 0 и b ≠ 0, то уравнение ах = b же имеет корней, так как равенство 0x = b не является верным ни при каком х. Если а = 0 и b = 0, то любое значение х является корнем уравнения, так как равенство 0х = 0 верно при любом х.

Решение многих уравнений сводится к решению линейных уравнений.

Пример. Решим уравнение 4(х + 7) = 3 – х.

► Раскроем скобки: 4х + 28 = 3 – х.

Перенесём слагаемое –х в левую часть уравнения, а слагаемое 28 в правую часть, изменив при этом их знаки: 4х + х = 3 – 28.

Приведём подобные слагаемые: 5х = –25.

Разделим обе части уравнения на 5: х = –5.

Применяя свойства уравнений и выполняя тождественные преобразования, мы последовательно заменяли одно уравнение другим, равносильным ему. Значит, корнем уравнения 4(х + 7) = 3 – х является число –5.

В этом примере исходное уравнение свелось к равносильному линейному уравнению, в котором коэффициент при переменной отличен от нуля.

Если при решении уравнения мы придём к равносильному ему линейному уравнению вида 0х = b, то в этом случае либо исходное уравнение не имеет корней, либо его корнем является любое число.

Решим уравнение 2х + 5 = 2 (х + 6):   2х + 5 = 2х + 12,   2х – 2х = 12 – 5,   0х = 7.

Полученное уравнение не имеет корней. Значит, и уравнение 2х + 5 = 2 (х + 6) не имеет корней.

Уравнение 3 (х + 2) + х = 6 + 4х сводится к уравнению 0x = 0, корнем которого является любое число. Следовательно, корнем уравнения 3(х + 2) + х = 6 + 4х является любое число.

Упражнения

8. Решение задач с помощью уравнений

При решении задач с помощью уравнений поступают следующим образом:

Задача 1. В корзине было в 2 раза меньше яблок, чем в ящике. После того как из корзины переложили в ящик 10 яблок, в ящике их стало в 5 раз больше, чем в корзине. Сколько яблок было в корзине и сколько в ящике?

► Пусть в корзине было х яблок, тогда в ящике было 2х яблок. После того как из корзины переложили в ящик 10 яблок, в корзине стало х – 10 яблок, а в ящике стало 2х + 10 яблок. По условию задачи в ящике стало в 5 раз больше яблок, чем в корзине. Значит, 5(х – 10) = 2х + 10.

Решим составленное уравнение: 5х – 50 = 2х + 10,   5х – 2х = 10 + 50,   3х = 60,   х = 20.

Следовательно, в корзине было 20 яблок. Так как 2х = 2 • 20 = 40, то в ящике было 40 яблок.
Ответ: 20 яблок и 40 яблок.

Задача 2. Предназначенные для посадки 78 саженцев смородины решили распределить между тремя бригадами так, чтобы первой бригаде досталось саженцев в 2 раза меньше, чем второй, а третьей – на 12 саженцев больше, чем первой. Сколько саженцев надо выделить первой бригаде?

► Пусть первой бригаде решили выделить х саженцев. Тогда второй следует выделить 2х саженцев, а третьей х + 12 саженцев. Общее число саженцев х + 2х + (х + 12), что по условию задачи равно 78.

Значит: х + 2х + (х + 12) = 78.

Решим полученное уравнение: х + 2х + х + 12 = 78,   4х = 78 – 12,   4х = 66,   х = 16,5.

По смыслу задачи значение х должно быть натуральным числом, а корень уравнения – дробное число. Значит, распределить саженцы указанным способом нельзя.
Ответ: Такое распределение саженцев невозможно.

Упражнения
Контрольные вопросы и задания
  1. Сформулируйте определение корня уравнения. Является ли число 7 корнем уравнения: 6х = 42; 0х = 11; (16 – 2 • 8)х = 0?
  2. Что значит решить уравнение? Решите уравнение: 6х = –12; x – 2x • 6 = 0; 5х – 4х = 6 + х.
  3. Какие уравнения называются равносильными? Сформулируйте свойства уравнений. Приведите пример уравнения, равносильного уравнению: 5х – 1 = 3; 0,2х = 1,1; 3х – 4х + 6 = 0.
  4. Дайте определение линейного уравнения с одной переменной. Приведите примеры.
  5. В каком случае уравнение ах = b имеет единственный корень; имеет бесконечно много корней; не имеет корней? Приведите примеры.

Дополнительные упражнения к параграфу 3

 


Вы смотрели ознакомительную версию с цитатами из учебника: Алгебра. 7 класс. Учебник / Макарычев Ю.Н., Миндюк Н.Г., Нешков К.И. и др.; под ред. С.А. Теляковского — 8-е издание. М.: Просвещение (2018). ГЛАВА I. § 3. УРАВНЕНИЯ С ОДНОЙ ПЕРЕМЕННОЙ.  (6. Уравнение и его корни. 7. Линейное уравнение с одной переменной. 8. Решение задач с помощью уравнений. Упражнения №№ 111 — 166. Контрольные вопросы и задания. Дополнительные упражнения №№ 233 — 252 к параграфу 3). Цитаты из учебника использованы в учебных целях.

Линейное уравнение с одной переменной. 7-й класс

Урок № 1.

Тип урока: закрепление пройденного материала.

Цели урока:

Образовательные:

  • формирование навыка решения уравнения с одним неизвестным сведением его
    к линейному уравнению с помощью свойств равносильности.

Развивающие:

  • формирование ясности и точности мысли, логического мышления, элементов
    алгоритмической культуры;
  • развитие математической речи;
  • развитие внимания, памяти;
  • формирование навыков само и взаимопроверки.

Воспитательные:

  • формирование волевые качества;
  • формирование коммуникабельность;
  • выработка объективной оценки своих достижений;
  • формирование ответственности.

Оборудование: интерактивная доска, доска для фломастеров, карточки с
заданиями для самостоятельной работы, карточки для коррекции знаний для
слабоуспевающих учащихся, учебник, рабочая тетрадь, тетрадь для домашних работ,
тетрадь для самостоятельных работ.

Ход урока

1. Организационный момент – 1мин.

Поприветствовать учащихся, проверить их готовность к уроку, объявить тему
урока и цель урока.

2. Проверка домашнего задания – 4 мин.

Учащиеся проверяют домашнюю работу, решение которой выведено с обратной
стороны доски одним из учащихся.

3. Устная работа– 6 мин.

(1) Пока идет устный счет, слабоуспевающие учащиеся получают карточку для
коррекции знаний
и выполняют 1), 2), 4) и 6) задания по образцу. (См.
Приложение 1.
)

Карточка для коррекции знаний.

(2) Для остальных учащихся задания проецируются на интерактивную доску: (См.
Презентацию: Слайд 2)

  1. Вместо звездочки поставь знак “+” или “–”, а вместо точек – числа:
    а) (*5)+(*7) = 2;

    б) (*8) – (*8) = (*4)–12;

    в) (*9) + (*4) = –5;

    г) (–15) – (*…) = 0;

    д) (*8) + (*…) = –12;

    е) (*10) – (*…) = 12.

  2. Составь уравнения, равносильные уравнению:

    а) х – 7 = 5;

    б) 2х – 4 = 0;

    в) х –11 = х – 7;

    г) 2(х –12) = 2х – 24.

3. Логическая задача: Вика, Наташа и Лена в магазине купили капусту,
яблоки и морковь. Все купили разные продукты. Вика купила овощ, Наташа – яблоки
или морковь, Лена купила не овощ. Кто что купил? (Один из учащихся,
выполнивший задание выходит к доске и заполняет таблицу.) (Слайд 3)


ВикаНаташаЛена
К


Я


М


  Заполнить таблицу





ВикаНаташаЛена
К+
Я+
М+

 Ответ

(Учащиеся используют пластиковые листы и фломастеры. )

4. Обобщение умения решать уравнения сведением их к линейному уравнению –9
мин.

Коллективная работа с классом. (Слайд 4)

Решим уравнение


12 – (4х – 18) = (36 + 5х) + (28 – 6х). (1)

для этого выполним следующие преобразования:

1. Раскроем скобки. Если перед скобками стоит знак “плюс”, то скобки
можно опустить, сохранив знак каждого слагаемого, заключенного в скобки. Если
перед скобками стоит знак “минус”, то скобки можно опустить, изменив знак
каждого слагаемого, заключенного в скобки:


12 – 4х + 18 = 36 + 5х + 28 – 6х. (2)

Уравнения (2) и (1) равносильны:

2. Перенесем с противоположными знаками неизвестные члены так, чтобы
они были только в одной части уравнения (или в левой, или в правой).
Одновременно перенесем известные члены с противоположными знаками так, чтобы они
были только в другой части уравнения.

Например, перенесем с противоположными знаками неизвестные члены в левую, а
известные – в правую часть уравнения, тогда получим уравнение


– 4х – 5х + 6х = 36 + 28 – 18 — 12, (3)

равносильное уравнению (2), а следовательно, и уравнению (1).

3. Приведем подобные слагаемые:


–3х = 34. (4)

Уравнение (4) равносильно уравнению (3), а следовательно, и
уравнению (1).

4. Разделим обе части уравнения (4) на коэффициент при
неизвестном.

Полученное уравнение х =
будет
равносильно уравнению (4), а следовательно, и уравнениям (3), (2), (1)

Поэтому корнем уравнения (1) будет число

По этой схеме (алгоритму) решаем уравнения на сегодняшнем уроке:

  1. Раскрыть скобки.
  2. Собрать члены, содержащие неизвестные, в одной части уравнения, а
    остальные члены в другой.
  3. Привести подобные члены.
  4. Разделить обе части уравнения на коэффициент при неизвестном.


Примечание: следует отметить, что приведенная схема не является
обязательной, так как часто встречаются уравнения, для решения которых некоторые
из указанных этапов оказываются ненужными. При решении же других уравнений
бывает проще отступить от этой схемы, как, например, в уравнении:


7(х – 2) = 42.

5. Тренировочные упражнения – 8 мин.

№ № 132(а, г), 135(а, г), 138(б, г) – с комментарием и записью на
доске.

6. Самостоятельная работа – 14 мин. (выполняется в тетрадях для
самостоятельных работ с последующей взаимопроверкой проверкой; ответы будут
отображены на интерактивной доске)

Перед самостоятельной работой учащимся будет предложено задание на
сообразительность – 2 мин.

Не отрывая карандаша от бумаги и не проходя дважды по одному и тому же
участку линии, начертите распечатанное письмо. (Слайд 5)

(Учащиеся используют пластиковые листы и фломастеры.)

1. Решить уравнения (на карточках) (См. Приложение 2)


Дополнительное задание № 135 (б, в).

7. Подведение итогов урока – 1 мин.

Алгоритм сведения уравнения к линейному уравнению.

8. Сообщение домашнего задания – 2 мин.

п.6, № № 136 (а-г), 240 (а), 243(а, б), 224 (Разъяснить содержание
домашнего задания).

Урок № 2.

Цели урока:

Образовательные:

  • повторение правил, систематизация, углубление и расширение ЗУНов
    учащихся по решению линейных уравнений;
  • формирование умения применять полученные знания при решении уравнений
    различными способами.

Развивающие:

  • развитие интеллектуальных умений: анализа алгоритма решения уравнения,
    логического мышления при построении алгоритма решения уравнения,
    вариативности выбора способа решения, систематизации уравнений по способам
    решения;
  • развитие математической речи;
  • развитие зрительной памяти.

Воспитательные:

  • воспитание познавательной активности;
  • формирование навыков самоконтроля, взаимоконтроля и самооценки;
  • воспитание чувства ответственности, взаимопомощи;
  • привитие аккуратности, математической грамотности;
  • воспитание чувства товарищества, вежливости, дисциплинированности,
    ответственности;
  • Здоровьесбережение.

а) образовательная: повторение правил, систематизация, углубление и
расширение ЗУНов учащихся по решению линейных уравнений;

б) развивающая: развитие гибкости мышления, памяти, внимания и
сообразительности;

в) воспитательная: привитие интереса к предмету и к истории родного края.

Оборудование: интерактивная доска, сигнальные карточки (зеленая и
красная), листы с тестовой работой, учебник, рабочая тетрадь, тетрадь для
домашних работ, тетрадь для самостоятельных работ.

Форма работы: индивидуальная, коллективная.

Ход урока

1. Организационный момент – 1мин.

Поприветствовать учащихся, проверить их готовность к уроку, объявить тему
урока и цель урока.

2. Устная работа – 10 мин.

(Задания для устного счета выводятся на интерактивную доску.)
(Слайд 6)

1) Решите задачи:

а) Мама старше дочери на 22 года. Сколько лет маме, если им вместе 46 лет

б) В семье трое братьев и каждый следующий младше предыдущего в два раза. Вместе
всем братьям 21 год. Сколько лет каждому?

2) Решите уравнения: (Пояснить)


;

 

 

 

 

 

 

 

 

 

Какие из данных уравнений являются линейными?

(Во время устного счета учащиеся используют сигнальные карточки: зеленую и
красную)

3) Проверьте, правильно ли решено уравнение, если нет, то найди ошибки.
(Слайд 7)


4 · (х – 5) = 12 – х

4х – 5 = 12 – х

4х + х = 12 – 5

5х = 7 /:5

х = 1,4
Желающий выходит к интерактивной доске

 исправить ошибки

 

4) Пояснить задания из домашней работы, вызвавшие затруднение.

3. Выполнение упражнений – 10 мин. (Слайд 8)

(1) Какому неравенству удовлетворяет корень уравнения:

4 – 5х = 5


а) x > 1;

б) x < 0;

в) x > 0;

г) x < –1.

(2) При каком значении выражении у значение выражения 2у – 4
в 5 раз меньше значения выражения 5у – 10?

(3) При каком значении k уравнение kx – 9 = 0
имеет корень равный – 2?

Посмотри и запомни (7 секунд). (Слайд 9)

Через 30 секунд учащиеся воспроизводят рисунок на пластиковых листах.

4. Физкультминутка – 1,5 мин.


Упражнение для глаз и для рук

(Учащиеся смотрят и повторяют упражнения, которые проецируются на
интерактивную доску.)

5. Самостоятельная тестовая работа – 15 мин.

(Учащиеся выполняют тестовую работу в тетрадях для самостоятельных работ,
дублируя ответы в рабочих тетрадях. Сдав тесты, учащиеся сверяют ответы с
ответами, отображенными на доске)

Учащиеся, справившиеся с работой раньше всех, помогают слабоуспевающим
учащимся.

(См. Приложение 3)


6. Подведение итогов урока – 2 мин.


– Какое уравнение с одной переменной называется линейным?

– Что называется корнем уравнения?

– Что значит “решить уравнение”?

– Сколько корней может иметь уравнение?

7. Сообщение домашнего задания. – 1 мин.

п.6, № № 294(а, б),244, 241(а, в), 240(г) – Уровень А, В

п.6, № № 244, 241(б, в), 243(в),239, 237– Уровень С

(Разъяснить содержание домашнего задания.)

8. Рефлексия – 0,5 мин.


– Вы довольны своей работой на уроке?

– Какой вид деятельности вам понравился больше всего на уроке.

Литература:

  1. Алгебра 7. / Ю.Н. Макарычев, Н.Г. Миндюк, К.И. Пешков, С.В. Суворова.
    Под редакцией С.А. Теляковского. / М.: Просвещение, 1989 – 2006.
  2. Сборник тестовых заданий для тематического и итогового контроля. Алгебра
    7 класс/ Гусева И.Л., Пушкин С.А., Рыбакова Н.В.. Общая ред.:
    Татур А.О.
    – М.: “Интеллект-Центр” 2009 – 160 с.
  3. Поурочное планирование по алгебре. / Т.Н.Ерина. Пособие для учителей /М:
    Изд. “Экзамен”, 2008. – 302,[2] с.
  4. Карточки для коррекции знаний по математике для 7 класса./ Левитас
    Г.Г.
    /М.: Илекса, 2000. – 56 с.

Урок 51. обобщение и систематизация знаний по теме «линейные уравнения» — Алгебра — 7 класс

Алгебра

7 класс

Урок № 51

Обобщение и систематизация знаний по теме: «Линейные уравнения»

Перечень вопросов, рассматриваемых в теме:

Связь понятий: «линейное уравнение», система линейных уравнений», «линейная функция», «решение линейного уравнения», «решение системы линейных уравнений».

Способы решения систем линейных уравнений.

Тезаурус:

Уравнение вида ax = b, (где x – переменная, a, b – некоторые числа), называется линейным уравнением с одной переменной.

Система вида

(где x, y – переменные, ai, bi, ci – некоторые числа) называется системой линейных уравнений с двумя переменными.

Основная литература:

1. Никольский С. М. Алгебра: 7 класс. // Никольский С. М., Потапов М. К., Решетников Н. Н., Шевкин А. В. – М.: Просвещение, 2017. – 287 с.

Дополнительная литература:

1. Чулков П. В. Алгебра: тематические тесты 7 класс. // Чулков П. В. – М.: Просвещение, 2014 – 95 с.

2. Потапов М. К. Алгебра: дидактические материалы 7 класс. // Потапов М. К., Шевкин А. В. – М.: Просвещение, 2017. – 96 с.

3. Потапов М. К. Рабочая тетрадь по алгебре 7 класс: к учебнику С. М. Никольского и др. «Алгебра: 7 класс». 1, 2 ч. // Потапов М. К., Шевкин А. В. – М.: Просвещение, 2017. – 160 с.

Теоретический материал для самостоятельного изучения

Уравнение вида (где x – переменная, a, b – некоторые числа) называется линейным уравнением с одной переменной.

a и b – коэффициенты линейного уравнения.

К уравнению такого вида можно привести уравнение, которое включает в себя переменную в первой степени.

Пример:

Для того, чтобы привести уравнение к виду ax = b, нужно его преобразовать.

Пример.

Рассмотрим уравнение.

Раскроем скобки и приведём подобные слагаемые:

В зависимости от значения коэффициентов, линейное уравнение может иметь либо один корень, либо ни одного корня, либо бесконечно много корней.

Если уравнение включает в себя две переменные в первой степени, получаем линейное уравнение с двумя переменными:

Можно из данного равенства выразить переменную y.

Получим уравнение линейной функции:

Её графиком является прямая. Таким образом, графиком линейного уравнения с двумя переменными является прямая, угловой коэффициент которой равен:

На прямой лежит бесконечно много точек, поэтому линейное уравнение с двумя переменными имеет бесконечно много решений. Все пары точек, координаты которых удовлетворяют уравнению:

Или координаты точек, лежащих на прямой, соответствующей уравнению.

Рассмотрим два линейных уравнения с двумя переменными и составим из них систему.

Геометрической интерпретацией решения системы двух уравнений с двумя переменными является точка пересечения прямых (если она есть).

Две прямые:

1) могут пересекаться (иметь одну общую точку), если их угловые коэффициенты не равны. В этом случае система имеет единственное решение.

Две прямые пересекаются, если:

– система имеет единственное решение;

2) могут быть параллельными (не иметь ни одной общей точки), если их угловые коэффициенты равны, а свободные коэффициенты не равны. В этом случае система не имеет решений.

Две прямые параллельны, если:

– система не имеет решений.

3) могут совпадать (иметь бесконечно много общих точек), если их угловые коэффициенты и свободные коэффициенты равны. В этом случае система имеет бесконечно много решений.

Две прямые совпадают, если:

Система имеет бесконечно много решений.

Для системы линейных уравнений могут быть использованы разные способы решения: алгебраический, в рамках которого рассматривается способ подстановки и способ алгебраического сложения. Или графический метод.

Рассмотрим пример.

Заметим, что и первое, и второе уравнения включают в себя выражение (5x – 2y)

Во втором уравнении оно выражено. Его и подставим в первое уравнение:

Теперь первое уравнение зависит только от одной переменной x.

Подставим найденное значение во второе уравнение и найдём значение y:

Ответ:

Текст для углублённого изучения.

Одним из простейших уравнений с параметром является линейное уравнение.

Рассмотрим уравнение с параметром:

a(a — 2)x = a2 — 4

Решение:

Рассмотрим коэффициент при переменной x.

Если: a(a – 2) ≠ 0, то есть уравнение имеет единственное решение.

Рассмотрим те значения параметра a, при которых a(a – 2) = 0

Пусть a = 0, тогда получим уравнение: 0 · x = –4. Это уравнение решений не имеет.

Пусть a = 2, тогда получим уравнение: 0 · x = 0. Это уравнение имеет бесконечно много решений.

Запишем ответ:

При a = 0 уравнение решений не имеет.

При a = 2 уравнение имеет бесконечно много решений.

Разбор решения заданий тренировочного модуля

Задача 1.

Рассортируйте уравнения по количеству их корней:

3x – 2(x + 5) = 6x – 12(2 – x)

15(1 – x) + 3 = 7 – 4x – 11(x – 1)

-5(2x + 4) = 5 – 10x

Решение.

Рассмотрим первое уравнение. Раскроем скобки:

3x – 2(x + 5) = 6x – 12(2 – x)

3х – 2х – 10 = 6х – 24 + 12х

Коэффициент при переменной не обратится в 0. Поэтому уравнение имеет единственное решение.

Рассмотрим второе уравнение. Раскроем скобки:

15(1 – x) + 3 = 7 – 4x – 11(x – 1)

15 – 15х + 3 = 7 – 4х – 11х + 11

18 – 15х = 18 – 15х

После преобразований получим уравнение 0x = 0, которое имеет бесконечно много корней.

Рассмотрим третье уравнение. Раскроем скобки:

-5(2x + 4) = 5 – 10x

-10х – 20 = 5 – 10х

Получим уравнение 0х = 25, которое не имеет решений.

Задача 2.

Выберите значения параметра, при каждом из которых уравнение не имеет решений:

Решение.

Количество решений линейного уравнения зависит от коэффициента при переменной. Рассмотрим его.

Приравняем его к нулю: a(a2 – 9) = 0

Найдем значения параметра:

a = 0

a = 3

a = –3

При каждом из этих значений параметра уравнение имеет вид:

0 · x = k, где k ≠ 0

Поэтому при каждом из этих значений параметра уравнение решений не имеет.

7 класс. Линейное уравнение с одной переменной. | План-конспект урока по алгебре (7 класс):

Название урока и класс:

Линейное уравнение с одной переменной. 7 класс.

Цели:

Цель урока:

1. Познакомить учащихся с тем, что такое линейное уравнение с одной переменной, что называется корнем уравнения, как решать уравнения с помощью алгоритма решения уравнений с одной переменной;

2. Способствовать воспитанию интереса к математике, упорства в достижении поставленной цели, трудолюбия, аккуратности.

3.Формировать умение работать с образовательной платформой Учи.ру

Ожидаемые результаты

      Предметные: знать понятие линейного уравнения, равносильности уравнений, корней уравнения; уметь применять полученные знания при решении упражнений

       Метапредметные: уметь устанавливать причинно – следственные связи, строить логическое рассуждение, умозаключение. Умение действовать в соответствии с предложенным алгоритмом.

       Личностные: готовность к жизненному и личностному самоопределению, знания моральных норм, умения выделять нравственный аспект поведения и соотносить поступки и события с принятыми этическими нормами, ориентация в жизненных ролях и межличностных отношениях (формируются во время выполнения заданий, в которых школьникам предлагается дать собственную оценку)

       Регулятивные: уметь поставить учебную цель, задачу на основе того, что уже известно и усвоено; уметь планировать последовательность своих действий для достижения конечного результата.

        Коммуникативные: планирование учебного сотрудничества с учителем и сверстниками; постановка вопросов.

Этапы урока:

Время (мин.)

Деятельность учителя

Ссылки на карточки Учи.ру

Организационный момент

2

Здравствуйте, ребята!Проверьте свою готовность к уроку, проверьте принадлежности ,чтоб на парте у каждого были учебник ,тетрадь ,дневник, письменные принадлежности.

Актуализация знаний

5

Д.з к доске два человека, остальные работают на повторении. Прежде чем перейдём к изучению нового материала, давайте с вами немного повторим ( привидение подобных слагаемых, раскрытие скобок) . Посмотрите на доску, по очереди раскроем скобки, приведём подобные слагаемые:

a-(3+b)=

c-b(3c-c+2b)=

3d+4(c-3b+d)=

https://uchi.ru/teachers/groups/1013373/subjects/1/course_programs/7/lessons/9999

Самоопределение к деятельности Введение понятия.

5

примеры линейных уравнений с одной переменной. ( учащиеся говорят, записываю на доске) 3x=12, 5y=10, 2a+7=0….

_ Как вы думаете, что значит решить уравнение? (ответ уч-ся: решить уравнение значит найти все те значения переменных, при которых уравнение обращается в верное равенство) Молодцы! Так вот каждое значение переменной называют корнем уравнения. Так какие корни имеют ,написанные нами на доске уравнения? (ответ учащихся: 3x=12, имеет корень x=4,т.к. 3*4=12 и т.д.)

 определение линейному уравнению с одной переменной: Линейным уравнением с одной переменной x наз-ся уравнение вида ax+b=0 ,где a и b любые числа( коэффициенты). Если a=0 , b=0 , т.е. уравнение имеет вид 0x+0=0 , то корнем уравнения является любое число (бесконечное множество ) . Если a=0 ,b=0, уравнение имеет вид 0x+b=0., то ни одно число этому уравнению не удовлетворяет, т.е. корней нет.

Рассмотрим наиболее распространённый вид уравнения, когда a=0 ,

1)​ ax+b=0 = ax=-b (слагаемое перенесли вправо с противоположным знаком)

2)​ x=- b/a.

Фактически мы выработали определённый порядок действий, т.е. алгоритм .( стр. 12-13

 учебника )

Работа по теме урока

7

Алгоритм решения линейного уравнения ax+b=0 в случае, когда a=0

1.​ Преобразовать уравнение к виду ax=-b.

2.​ Записать корень уравнения в виде x=( — b) : a , или , что тоже самое , x=-b/a.

А как же быть ,если уравнение имеет такой вид, например: 2x-2=10-x? (пробуют ответить уч-ся) . Рассуждаем так: Два выражения равны тогда и только тогда, когда их разность равна 0. т.е. ( 2x-2)- (10-x)=0. Что делам дальше?(ответ уч-ся: Раскрываем скобки, приводим подобные слагаемые) К доске идёт ученик, остальные записывают в тетрадях.

Нашли x=4. А можем мы всё это решение обобщить в алгоритм? Конечно. ( стр.21 учебника)

Алгоритм решения уравнения ax+b=cx=d ( a=c)

1.​ Перенести все члены уравнения из правой части в левую с противоположными знаками.

2.​ Привести в левой части подобные слагаемые , в результате чего получится уравнение вида kx+m=0, где k=0.

3.​ Преобразовать уравнение к виду kx=-m и записать его корень : x=-m/k.

Работа с заданиями от Учи.ру на интерактивной  доске

https://uchi.ru/teachers/groups/1013373/subjects/1/course_programs/7/lessons/10006

Закрепление материала

7

Учебник стр 14 № 2.5

Физкультминутка

2

Мини Тест

9

Работа с заданиями от Учи.ру на интерактивной  доске

https://uchi.ru/teachers/groups/1013373/subjects/1/course_programs/7/lessons/10008

Подведение итогов урока

3

Итак, ребята, что на уроке вы узнали нового?

( что наз-ся уравнением с одной переменной) ,

А что называется коэффициентом?( число при неизвестном ,или переменной)

Что есть корень уравнения? ( Значение переменной, при котором уравнение переходит в верное равенство)

Чему научились ? ( решать линейные уравнения с помощью алгоритма).

 

Домашнее задание

Учебник стр.14,  №2.6

https://uchi.ru/teachers/groups/1013373/subjects/1/course_programs/7/lessons/10007

Линейное уравнение с одной переменной 25у — 10 = 0. 7 класс

Линейное уравнение с
одной переменной
25у — 10 = 0
1
Одной
из самых
простых и
важных
математических
моделей реальных
ситуаций есть
линейные
уравнения с одной
переменной
3х = 12
5у + 10 = 0
2,1а -7 = 0
45+36х= -23
(23+12а)= 124
2

3. Запомни! При решении уравнения нужно сделать проверку.

Решить линейное
уравнение с одной
переменной – это
значит найти те
значения
переменной,
при каждом из которых
уравнение обращается
в верное числовое
равенство.
3
Найдём корень уравнения:
Решили уравнение – нашли те
значения переменной, при
котором уравнение
обращается в верное числовое
равенство.
4
Не решая уравнений,
проверь, какое из чисел
является корнем
уравнения.
87 + (32 – х) = 105
5
Решить уравнение – это
Решим
уравнение:
значит
найти
все его
корни или доказать, что
их нет
+ у = 31 +
+ у = 46
y = 46 -35
6

7. Тип 3.Уравнение имеет бесконечное множество решений? Решите уравнения и проведите классификацию уравнений по трем типам.

Тип 1.Уравнение
имеет решение?
23= 245+(Х- 12)
34+12,6Х=23-12,6Х
14,8=С -12,89С
Тип 2.Уравнение не
имеет решение?
23х=235 + 23(Х+10)
12,5+5,6=У-67
5,9в-1,2в=4,7в
Тип 3.Уравнение имеет бесконечное
множество решений?
Решите уравнения и проведите
классификацию уравнений по трем типам.
7
Уравнения, которые имеют одни и
те же корни, называют
равносильными.
x — 5x + 6 = 0 и
2
(х — 2)(х — 3) = 0
Равносильные уравнения
Каждое уравнение имеет одни и
те же корни
х₁ = 2 х₂ = 3
8
1. Если в уравнении перенести слагаемое из одной
части в другую, изменив его знак, то получится
равносильное уравнение.
2. Если обе части уравнения умножить или
разделить на число (не равное нулю), то
получится равносильное
уравнение.
9
(у — 35) + 12 = 32;
Решение уравнений состоит в постепенной замене
более простыми равносильными уравнениями
Решение.
у — 35 + 12 = 32;
у – 23 = 32;
у = 32 + 23;
у = 55;
(55 — 35) + 12 = 32;
30 + 12 = 32;
32 = 32.
Ответ: 55.
10
х – переменная входит в уравнение
обязательно в первой степени.
(45 — у) + 18 = 58
3х² + 6х + 7 = 0
11
2(3х — 1) = 4(х + 3)
Решение уравнений состоит в постепенной замене
более простыми равносильными уравнениями.
Приведем к стандартному виду:
2(3х — 1) = 4(х + 3)
6х – 2 = 4х + 12
6х – 4х = 2 + 12
х = 14 : 2
х=7
12
2(3х — 1) = 4(х + 3) – 14 + 2х
Приведем к стандартному виду:
2(3х — 1) = 4(х + 3) – 14 + 2х
6х – 2 = 4х + 12 – 14 + 2х
6х – 4x — 2х = 2 + 12 – 14
(а = 0, b = 0)
При подстановке любого значения х получаем
верное числовое равенство:
0=0
x – любое число
13

14. Ответим на вопросы?

1.
2.
3.
4.
5.
6.
7.
8.
Что называется
уравнением?
Что называется
корнем уравнения?
Сколько корней
может иметь
уравнение?
Какие уравнения
называются
равносильными?
Сформулируйте
основные свойства
уравнений.
Стандартный вид
линейного уравнения.
Какое уравнение
называется
линейным?
Ответим на вопросы?
14

15. Спасибо за внимание!

15

Решение линейных уравнений с одной переменной

Линейные уравнения с одной переменной — это уравнения, в которых переменная имеет показатель степени 1, который обычно не отображается (это понятно). Примером может быть что-то вроде \ (12x = x — 5 \). Для решения линейных уравнений есть одна основная цель: изолировать переменную . В этом уроке мы рассмотрим, как это делается, на нескольких примерах.

Содержание

  1. Примеры решения одношаговых уравнений
  2. Примеры решения двухэтапных уравнений
  3. Примеры уравнений, в которых сначала необходимо упростить
  4. Бесконечно много или нет решений
  5. Сводка

объявление

Примеры решения одношаговых линейных уравнений

После всей вашей тяжелой работы над решением уравнения вы знаете, что хотите получить окончательный ответ, например \ (x = 5 \) или \ (y = 1 \).В обоих этих случаях переменная изолирована или сама по себе.

Итак, нам нужно выяснить, как изолировать переменную. Как мы это сделаем, зависит от самого уравнения! Если его на что-то умножили, поделим. Если к нему что-то добавили, мы вычтем. Поступая так, мы постепенно будем получать переменную сама по себе.

Давайте рассмотрим пример, чтобы увидеть, как это работает.

Пример

Решите уравнение: \ (4x = 8 \)

Решение

В этом примере 4 — это умножение на \ (x \).Следовательно, чтобы изолировать \ (x \), вы должны разделить эту сторону на 4. Делая это, вы должны помнить одно важное правило: что бы вы ни делали с одной стороной уравнения, вы должны делать с другой стороной. Итак, мы разделим обе стороны на 4.

\ (\ begin {align} 4x & = 8 \\ \ dfrac {4x} {\ color {red} {4}} & = \ dfrac {8} {\ color {red} {4}} \ end {align} \)

Упрощение:

\ (х = \ в коробке {2} \)

Вот и все, один шаг, и готово. (Вот почему подобные уравнения часто называют «одношаговыми» уравнениями)

Чек

Каждый раз, когда вы решаете линейные уравнения, вы всегда можете проверить свой ответ, подставив его обратно в уравнение.Если вы получите верное утверждение, значит, ответ правильный. Это не обязательно на 100% для каждой задачи, но это хорошая привычка, поэтому мы сделаем это для наших уравнений.

В этом примере наше исходное уравнение было \ (4x = 8 \). Чтобы проверить это, убедитесь, что верно следующее:

\ (\ begin {align} 4x & = 8 \\ 4 (2) & = 8 \\ 8 & = 8 \ end {align} \)

Это верное утверждение, поэтому наш ответ правильный.

Для любого уравнения любая операция, которую вы выполняете с одной стороной, должна выполняться и с другой стороной.

Давайте попробуем еще пару примеров, прежде чем переходить к более сложным уравнениям.

Пример

Решить: \ (3x = 12 \)

Решение

Поскольку \ (x \) умножается на 3, план состоит в том, чтобы разделить на 3 с обеих сторон:

\ (\ begin {align} 3x & = 12 \\ \ dfrac {3x} {\ color {red} {3}} & = \ dfrac {12} {\ color {red} {3}} \\ x & = \ в штучной упаковке {4} \ end {align} \)

Чек

Чтобы проверить наш ответ, мы позволим \ (x = 4 \) и подставим его обратно в уравнение:

\ (\ begin {align} 3x & = 12 \\ 3 (4) & = 12 \\ 12 & = 12 \ end {align} \)

Как и раньше, поскольку это истинное утверждение, мы знаем, что наш ответ правильный.

В следующем примере вместо умножения переменной на значение из переменной вычитается значение. Чтобы «отменить» это, мы добавим это значение обеим сторонам.

Пример

Решить: \ (y-9 = 21 \)

Решение

На этот раз из y вычитается 9. Итак, мы отменим это, добавив 9 к обеим сторонам.

\ (\ begin {align} y-9 & = 21 \\ y-9 \ color {red} {+ 9} & = 21 \ color {red} {+ 9} \\ y & = 30 \ end {align} \)

Далее мы рассмотрим то, что обычно называют «двухшаговыми» уравнениями.В этих уравнениях нам нужно будет отменить две операции, чтобы изолировать переменную.

Примеры двухшаговых уравнений

В каждом из приведенных выше примеров нужно было выполнить один шаг, прежде чем мы получили ответ. В следующих примерах вы увидите, как работать с уравнениями, которые вместо этого состоят из двух шагов. Если выполняется более одной операции, важно помнить порядок операций PEMDAS. Поскольку вы отменяете операции с \ (x \), вы будете работать «снаружи внутрь».Это легче понять, когда вы увидите это на примере.

Пример

Решить: \ (2x-7 = 13 \)

Решение

Обратите внимание на две операции, происходящие с \ (x \): он умножается на 2, а затем вычитается 7. Нам нужно будет их отменить. Но только \ (x \) умножается на 2, поэтому первым шагом будет прибавление 7 к обеим сторонам. Тогда мы можем разделить обе части на 2.

Добавление 7 к обеим сторонам:

\ (\ begin {align} 2x-7 & = 13 \\ 2x-7 \ color {red} {+ 7} & = 13 \ color {red} {+ 7} \\ 2x & = 20 \ end {align} \ )

Теперь разделите обе стороны на 2:

.

\ (\ begin {align} 2x & = 20 \\ \ dfrac {2x} {\ color {red} {2}} & = \ dfrac {20} {\ color {red} {2}} \\ x & = \ в штучной упаковке {10} \ end {align} \)

Чек

Как и в случае с более простыми задачами, вы можете проверить свой ответ, подставив свое значение \ (x \) обратно в исходное уравнение.

\ (\ begin {align} 2x-7 & = 13 \\ 2 (10) — 7 & = 13 \\ 13 & = 13 \ end {align} \)

Это правда, значит, у нас есть правильный ответ.

Давайте рассмотрим еще один пример с двумя шагами, прежде чем мы снова будем преодолевать трудности. Убедитесь, что вы понимаете каждый показанный шаг и также работаете над проблемой.

Пример

Решить: \ (5w + 2 = 9 \)

Решение

Как и выше, есть две операции: \ (w \) умножается на 5, а затем к нему прибавляется 2.Мы отменим их, сначала вычтя 2 с обеих сторон, а затем разделив на 5.

\ (\ begin {align} 5w + 2 & = 9 \\ 5w + 2 \ color {red} {- 2} & = 9 \ color {red} {- 2} \\ 5w & = 7 \\ \ dfrac { 5w} {\ color {red} {5}} & = \ dfrac {7} {\ color {red} {5}} \\ w = \ boxed {\ dfrac {7} {5}} \ end {align} \)

Дробь справа не может быть упрощена, так что это наш окончательный ответ.

Чек

Пусть \ (w = \ dfrac {7} {5} \). Тогда:

\ (\ begin {align} 5w + 2 & = 9 \\ 5 \ left (\ dfrac {7} {5} \ right) + 2 & = 9 \\ 7 + 2 & = 9 \\ 9 & = 9 \ конец {align} \)

Итак, мы снова получили правильный ответ!

Упрощение перед решением

В следующих примерах есть больше изменяемых терминов и, возможно, необходимо некоторое упрощение.В каждом случае шаги будут заключаться в том, чтобы сначала упростить обе стороны, а затем использовать то, что мы делали, чтобы изолировать переменную. Сначала мы подробно рассмотрим пример, чтобы увидеть, как все это работает.

Чтобы понять этот раздел, вам должно быть удобно комбинировать похожие термины.

Пример

Решить: \ (3x + 2 = 4x-1 \)

Решение

Поскольку обе части упрощены (нет скобок, которые нам нужно вычислять, и нет одинаковых членов для объединения), следующим шагом будет получение всех x на одной стороне уравнения и всех чисел на другой стороне.Применяется то же правило — что бы вы ни делали с одной стороной уравнения, вы должны делать и с другой стороной!

Можно перемещать \ (3x \) или \ (4x \). Предположим, вы переместили \ (4x \). Поскольку он положительный, вы должны вычесть его с обеих сторон:

\ (\ begin {align} 3x + 2 & = 4x-1 \\ 3x + 2 \ color {red} {- 4x} & = 4x-1 \ color {red} {- 4x} \\ -x + 2 & = -1 \ end {align} \)

Теперь уравнение выглядит так же, как и раньше. Следующим шагом будет вычитание 2 с обеих сторон:

\ (\ begin {align} -x + 2 \ color {red} {- 2} & = -1 \ color {red} {- 2} \\ — x = -3 \ end {align} \)

Наконец, поскольку \ (- x = -1x \) (это всегда верно), разделите обе стороны на \ (- 1 \):

\ (\ begin {align} \ dfrac {-x} {\ color {red} {- 1}} & = \ dfrac {-3} {\ color {red} {- 1}} \\ x & = 3 \ end {выровнять}\)

Чек

Вам следует воспользоваться моментом и убедиться, что следующее утверждение является верным:

\ (3 (3) + 2 = 4 (3) — 1 \)

В следующем примере нам нужно будет использовать свойство распределения перед решением.Здесь легко ошибиться, поэтому убедитесь, что вы распределили число перед круглыми скобками для всех терминов внутри.

Пример

Решить: \ (3 (x + 2) -1 = x-3 (x + 1) \)

Решение

Сначала разложите 3 и –3 и соберите одинаковые термины.

\ (\ begin {align} 3 (x + 2) -1 & = x-3 (x + 1) \\ 3x + 6-1 & = x-3x-3 \\ 3x + 5 & = — 2x-3 \ end {выровнять}\)

Теперь мы можем прибавить 2x к обеим сторонам. (Помните, что вы получите тот же ответ, если вместо этого вычтете 3x с обеих сторон)

\ (\ begin {align} 3x + 5 \ color {red} {+ 2x} & = — 2x-3 \ color {red} {+ 2x} \\ 5x + 5 & = -3 \ end {align} \)

Отсюда мы можем решить, как и с другими двухэтапными уравнениями.

\ (\ begin {align} 5x + 5 \ color {red} {- 5} & = — 3 \ color {red} {- 5} \\ 5x & = — 8 \\ \ dfrac {5x} {\ color { красный} {5}} & = \ dfrac {-8} {\ color {red} {5}} \\ x & = \ dfrac {-8} {5} \\ & = \ boxed {- \ dfrac {8 } {5}} \ end {align} \)

Чек

Это был сложный вопрос, поэтому не забудьте проверить свой ответ и убедиться, что не было допущено никаких ошибок. Для этого вы убедитесь, что следующее утверждение является верным:

\ (3 \ left (- \ dfrac {8} {5} +2 \ right) -1 = \ left (- \ dfrac {8} {5} \ right) -3 \ left (- \ dfrac {8} { 5} +1 \ вправо) \)

(Примечание: это работает, но вы должны быть очень осторожны с круглыми скобками!)

Бесконечно много решений и никаких решений

Бывают случаи, когда вы выполняете все эти шаги, и возникает действительно странное решение.Например, при решении уравнения \ (x + 2 = x + 2 \) с использованием описанных выше шагов в итоге получается \ (0 = 0 \). Это, конечно, правда, но что хорошего в этом?

Если вы получите подобное утверждение, это означает, что уравнение имеет бесконечно много решений. Любой \ (x \), о котором вы можете подумать, удовлетворял бы уравнению \ (x + 2 = x + 2 \). Подходящий ответ в этом случае — «бесконечно много решений».

Другая ситуация возникает, когда вы упрощаете уравнение до утверждения, которое никогда не бывает истинным, например \ (3 = 4 \) или \ (0 = 1 \).Это происходит с уравнением \ (x + 5 = x-7 \), которое приводит к \ (5 = -7 \), что, конечно, никогда не бывает истинным. Это означает, что никакое \ (x \) не удовлетворяет этому уравнению. Другими словами «нет решения». Итого:

  • Если вы получите утверждение, которое всегда истинно, например \ (5 = 5 \) или \ (0 = 0 \), то существует бесконечно много решений.
  • Если вы получаете утверждение, которое всегда ложно, например \ (10 ​​= 11 \) или \ (1 = 5 \), то решений нет.

реклама

Сводка

Решение линейных уравнений сводится к выделению переменной.В зависимости от уравнения это может занять всего один шаг или намного больше. Всегда проверяйте, нужно ли вам сначала упростить одну или обе стороны уравнения, и всегда проверяйте свой ответ.

Подпишитесь на нашу рассылку новостей!

Мы всегда публикуем новые бесплатные уроки и добавляем новые учебные пособия, руководства по калькуляторам и пакеты задач.

Подпишитесь, чтобы получать электронные письма (раз в пару или три недели) с информацией о новинках!

Связанные

Решение линейных уравнений

Решение основных линейных уравнений

EquationStatement, указывающий, что два алгебраических выражения равны.это утверждение, указывающее, что два алгебраических выражения равны. Линейное уравнение с одной переменной Уравнение, которое может быть записано в стандартной форме ax + b = 0, где a и b — действительные числа, а a ≠ 0., x — это уравнение, которое может быть записано в стандартная форма ax + b = 0, где a и b — действительные числа и a 0. Например,

3х − 12 = 0

Решение: Любое значение, которое может заменить переменную в уравнении для получения истинного утверждения.к линейному уравнению — это любое значение, которое может заменить переменную для получения истинного утверждения. Переменная в линейном уравнении 3x − 12 = 0 равна x , а решение — x = 4. Чтобы проверить это, замените значение 4 на x и убедитесь, что вы получили истинное утверждение.

3x − 12 = 03 (4) −12 = 012−12 = 00 = 0 ✓

В качестве альтернативы, когда уравнение равно константе, мы можем проверить решение, подставив значение переменной и показывая, что результат равен этой константе.В этом смысле мы говорим, что решения «удовлетворяют уравнению».

Пример 1

Является ли a = −12 решением −10a + 5 = 25?

Решение:

Напомним, что при оценке выражений рекомендуется сначала заменять все переменные круглыми скобками, а затем подставлять соответствующие значения. Используя круглые скобки, мы избегаем некоторых распространенных ошибок при определении порядка операций.

−10a + 5 = −10 (−12) + 5 = 5 + 5 = 10 ≠ 25 ✗

Ответ: Нет, a = −12 не удовлетворяет уравнению.

Разработка методов решения различных алгебраических уравнений — одна из наших основных целей в алгебре. В этом разделе рассматриваются основные методы, используемые для решения линейных уравнений с одной переменной. Начнем с определения эквивалентных уравнений Уравнения с одним и тем же набором решений. как уравнения с тем же набором решений.

3x − 5 = 16 3x = 21 x = 7} Эквивалентные уравнения

Здесь мы видим, что три линейных уравнения эквивалентны, потому что они имеют один и тот же набор решений, а именно {7}.Чтобы получить эквивалентные уравнения, используйте следующие свойства равенстваProperties, которые позволяют нам получать эквивалентные уравнения путем сложения, вычитания, умножения и деления обеих частей уравнения ненулевыми действительными числами. Даны алгебраические выражения A и B , где c — ненулевое число:

Дополнительное свойство равенства:

Если A = B, то A + c = B + c

Свойство вычитания равенства:

Если A = B, то A − c = B − c

Свойство умножения равенства:

Если A = B, то cA = cB

Разделение собственности равенства:

Если A = B, то Ac = Bc

Примечание: Следует тщательно избегать умножения или деления обеих частей уравнения на 0.Деление на 0 не определено, а умножение обеих частей на 0 приводит к уравнению 0 = 0.

Мы решаем алгебраические уравнения, выделяя переменную с коэффициентом 1. Если дано линейное уравнение вида ax + b = c, то мы можем решить его в два этапа. Во-первых, используйте соответствующее свойство равенства сложения или вычитания, чтобы изолировать переменный член. Затем изолируйте переменную, используя свойство равенства умножения или деления. Проверка решения в следующих примерах предоставляется читателю.

Пример 2

Решите: 7x − 2 = 19.

Решение:

7x − 2 = 197x − 2 + 2 = 19 + 2 Добавьте 2 к обеим сторонам. 7x = 217×7 = 217 Разделим обе стороны на 7.x = 3

Ответ: Решение — 3.

Пример 3

Решить: 56 = 8 + 12г.

Решение:

Если перед термином нет знака, это считается положительным.Другими словами, представьте себе это как 56 = + 8 + 12y. Поэтому мы начинаем с вычитания 8 по обе стороны от знака равенства.

56−8 = 8 + 12y − 848 = 12y4812 = 12y124 = y

Не имеет значения, с какой стороны мы выберем изолировать переменную, потому что свойство симметрии позволяет вам найти переменную по обе стороны от знака равенства, потому что x = 5 эквивалентно 5 = x. утверждает, что 4 = y эквивалентно y = 4.

Ответ: Решение — 4.

Пример 4

Решить: 53x + 2 = −8.

Решение:

Выделите переменный член, используя свойство сложения равенства, а затем умножьте обе части уравнения на обратную величину коэффициента 53.

53x + 2 = −853x + 2−2 = −8−2 Вычтем 2 с обеих сторон. 53x = −1035⋅53x = 35⋅ (−10) −2 Умножим обе стороны на 35,1x = 3⋅ (−2) х = −6

Ответ: Решение — 6.

Таким образом, чтобы сохранить эквивалентные уравнения, мы должны выполнить одну и ту же операцию с обеими сторонами уравнения.

Попробуй! Решите: 23x + 12 = −56.

Ответ: x = −2

Общие рекомендации по решению линейных уравнений

Обычно линейные уравнения не приводятся в стандартной форме, поэтому их решение требует дополнительных шагов. При решении линейных уравнений цель состоит в том, чтобы определить, какое значение, если оно есть, даст истинное утверждение при подстановке в исходное уравнение.Сделайте это, изолировав переменную, выполнив следующие шаги:

  • Шаг 1: Упростите обе части уравнения, используя порядок операций, и объедините все одинаковые члены с одной стороны от знака равенства.
  • Шаг 2: Используйте соответствующие свойства равенства, чтобы объединить одинаковые члены по разные стороны от знака равенства. Цель состоит в том, чтобы получить переменный член с одной стороны уравнения и постоянный член с другой.
  • Шаг 3: Разделите или умножьте, если необходимо, чтобы изолировать переменную.
  • Шаг 4: Проверьте, решает ли ответ исходное уравнение.

Мы часто будем сталкиваться с линейными уравнениями, в которых выражения по обе стороны от знака равенства могут быть упрощены. Если это так, то лучше сначала упростить каждую сторону, прежде чем решать. Обычно это включает в себя объединение одинаковых терминов.

Примечание: На данном этапе нашего изучения алгебры использование свойств равенства должно показаться обычным делом.Поэтому отображение этих шагов в этом тексте, обычно синем, становится необязательным.

Пример 5

Решите: −4a + 2 − a = 1.

Решение:

Сначала объедините одинаковые члены слева от знака равенства.

−4a + 2 − a = 1 Объедините одинаковые члены с одной стороны. −5a + 2 = 1 Вычтите 2 с обеих сторон. −5a = −1 Разделите обе части на −5. A = −1−5 = 15

Всегда используйте исходное уравнение, чтобы проверить правильность решения.

−4a + 2 − a = −4 (15) + 2−15 = −45 + 21⋅55−15 = −4 + ​​10 + 15 = 55 = 1 ✓

Ответ: Решение — 15.

Для линейного уравнения в форме ax + b = cx + d мы начинаем процесс решения, комбинируя одинаковые члены по разные стороны от знака равенства. Для этого используйте свойство равенства сложения или вычитания, чтобы разместить похожие термины на одной стороне, чтобы их можно было комбинировать. В оставшихся примерах проверка оставлена ​​на усмотрение читателя.

Пример 6

Решите: −2y − 3 = 5y + 11.

Решение:

Вычтем 5y с обеих сторон, чтобы мы могли объединить члены, включающие y в левой части.

−2y − 3−5y = 5y + 11−5y − 7y − 3 = 11

Отсюда решите, используя методы, разработанные ранее.

−7y − 3 = 11 Добавьте 3 к обеим сторонам. − 7y = 14y = 14−7 Разделите обе части на −7.y = −2

Ответ: Решение — 2.

Решение часто требует применения свойства распределения.

Пример 7

Решите: −12 (10x − 2) + 3 = 7 (1−2x).

Решение:

Сначала упростите линейные выражения по обе стороны от знака равенства.

−12 (10x − 2) + 3 = 7 (1−2x) Распределить. −5x + 1 + 3 = 7−14x Объединить одинаковые члены с одинаковой стороны. −5x + 4 = 7−14x Объединить одинаковые члены с противоположной стороны .9x = 3 Решить. X = 39 = 13

Ответ: Решение — 13.

Пример 8

Решите: 5 (3 − a) −2 (5−2a) = 3.

Решение:

Начните с применения свойства распределения.

5 (3 − a) −2 (5−2a) = 315−5a − 10 + 4a = 35 − a = 3 − a = −2

Здесь мы указываем, что −a эквивалентно −1a; поэтому мы решили разделить обе части уравнения на −1.

−a = −2−1a − 1 = −2−1a = 2

В качестве альтернативы мы можем умножить обе части −a = −2 на отрицательную единицу и получить тот же результат.

−a = −2 (−1) (- a) = (- 1) (- 2) a = 2

Ответ: Решение — 2.

Попробуй! Решите: 6−3 (4x − 1) = 4x − 7.

Ответ: x = 1

Есть три различных типа уравнений. До этого момента мы решали условные уравнения. Уравнения, которые верны для определенных значений. Это уравнения, которые верны для определенных значений.Идентичность — уравнение, истинное для всех возможных значений. — это уравнение, справедливое для всех возможных значений переменной. Например,
x = x идентичность
имеет набор решений, состоящий из всех действительных чисел, ℝ. Противоречие — уравнение, которое никогда не бывает истинным и не имеет решения. — это уравнение, которое никогда не бывает истинным и поэтому не имеет решений. Например,
x + 1 = x Противоречие
не имеет решения. Мы используем пустой набор Ø, чтобы указать, что решений нет.

Если конечным результатом решения уравнения является истинное утверждение, например 0 = 0, тогда уравнение является тождеством, а любое действительное число является решением.Если решение приводит к ложному утверждению, например 0 = 1, тогда уравнение противоречит и нет решения.

Пример 9

Решите: 4 (x + 5) + 6 = 2 (2x + 3).

Решение:

4 (x + 5) + 6 = 2 (2x + 3) 4x + 20 + 6 = 4x + 64x + 26 = 4x + 626 = 6 ✗

Решение приводит к ложному утверждению; следовательно, уравнение противоречит и не имеет решения.

Ответ: Ø

Пример 10

Решите: 3 (3y + 5) + 5 = 10 (y + 2) −y.

Решение:

3 (3y + 5) + 5 = 10 (y + 2) −y9y + 15 + 5 = 10y + 20 − y9y + 20 = 9y + 209y = 9y0 = 0 ✓

Решение приводит к верному утверждению; следовательно, уравнение является тождественным, а любое действительное число является решением.

Ответ: ℝ

Коэффициенты линейных уравнений могут быть любыми действительными числами, даже десятичными и дробными. В этом случае можно использовать свойство умножения равенства, чтобы очистить дробные коэффициенты и получить целочисленные коэффициенты за один шаг.Если даны дробные коэффициенты, умножьте обе части уравнения на наименьшее общее кратное знаменателей (ЖКД).

Пример 11

Решите: 13x + 15 = 15x − 1.

Решение:

Очистите дроби, умножив обе части на наименьшее общее кратное указанных знаменателей. В данном случае это ЖКД (3,5) = 15.

15⋅ (13x + 15) = 15⋅ (15x − 1) Умножаем обе части на 15.15⋅13x + 15⋅15 = 15⋅15x − 15⋅1 Упростить. 5x + 3 = 3x − 15 Решить. 2x = −18x = −182 = −9

Ответ: Решение — 9.

Важно знать, что этот метод работает только для уравнений. Не пытайтесь очищать дроби при упрощении выражений. Напоминание:

Выражение

Уравнение

12x + 53

12x + 53 = 0

Мы упрощаем выражения и решаем уравнения.Если вы умножите выражение на 6, вы решите проблему. Однако, если вы умножите обе части уравнения на 6, вы получите эквивалентное уравнение.

Неправильно

Правильно

12x + 53 ≠ 6⋅ (12x + 53) = 3x + 10 ✗

12x + 53 = 06⋅ (12x + 53) = 6⋅03x + 10 = 0 ✓

Приложения с линейными уравнениями

Алгебра упрощает процесс решения реальных проблем.Это делается с помощью букв для обозначения неизвестных, переформулирования проблем в форме уравнений и предложения систематических методов решения этих уравнений. Чтобы решить проблемы с использованием алгебры, сначала переведите формулировку проблемы в математические формулировки, которые описывают отношения между данной информацией и неизвестными. Обычно перевод в математические утверждения — трудный шаг в этом процессе. Ключ к переводу — внимательно прочитать проблему и определить определенные ключевые слова и фразы.

Ключевые слова

Перевод

Сумма , увеличенная, больше чем, плюс, прибавленная, всего

+

Разница , уменьшено, вычтено из, меньше, минус

Произведение , умноженное на, раз, дважды

Частное , деленное на, отношение, на

÷

Is , итого, результат

=

При переводе предложений в математические утверждения обязательно прочтите предложение несколько раз и проанализируйте ключевые слова и фразы.Важно сначала идентифицировать переменную « пусть x представляет… » и прописать словами, что такое неизвестная величина. Этот шаг не только делает нашу работу более читаемой, но и заставляет задуматься о том, что мы ищем.

Пример 12

Когда 6 вычитается из удвоенной суммы числа и 8, получается 5. Найдите число.

Решение:

Пусть n представляет неизвестное число.

Чтобы понять, почему мы включили круглые скобки в настройку, вы должны изучить структуру следующих двух предложений и их переводы:

«, удвоенная сумма числа и 8 »

2 (п + 8)

« сумма удвоенного числа и 8 »

2н + 8

Ключевым моментом было сосредоточиться на фразе « удвоить сумму », это побудило нас сгруппировать сумму в круглых скобках, а затем умножить на 2.После перевода предложения в математическое выражение мы решаем.

2 (n + 8) −6 = 52n + 16−6 = 52n + 10 = 52n = −5n = −52

Проверить.

2 (n + 8) −6 = 2 (−52 + 8) −6 = 2 (112) −6 = 11−6 = 5 ✓

Ответ: Число -52.

Ниже приведены общие рекомендации по настройке и решению проблем со словами.

  • Шаг 1: Прочтите проблему несколько раз, определите ключевые слова и фразы и систематизируйте данную информацию.
  • Шаг 2: Определите переменные, присвоив неизвестным величинам букву или выражение.
  • Шаг 3: Переведите и составьте алгебраическое уравнение, моделирующее проблему.
  • Шаг 4: Решите полученное алгебраическое уравнение.
  • Шаг 5: Наконец, ответьте на вопрос в форме предложения и убедитесь, что он имеет смысл (отметьте его).

На данный момент создайте все свои уравнения, используя только одну переменную.Избегайте двух переменных, ища взаимосвязь между неизвестными.

Пример 13

Прямоугольник имеет периметр 92 метра. Длина на 2 метра меньше ширины в 3 раза. Найдите размеры прямоугольника.

Решение:

Предложение « Длина составляет на 2 метра меньше , чем в 3 раза больше ширины », дает нам взаимосвязь между двумя переменными.

Пусть w представляет ширину прямоугольника.

Пусть 3w − 2 представляет длину.

Предложение « Прямоугольник имеет периметр размером 92 метров » предполагает алгебраическую установку. Подставьте 92 вместо периметра и выражение 3w − 2 для длины в соответствующую формулу:

P = 2l + 2w ↓ ↓ 92 = 2 (3w − 2) + 2w

После того, как вы составили алгебраическое уравнение с одной переменной, найдите ширину w .

92 = 2 (3w − 2) + 2wDistribute.92 = 6w − 4 + 2w Объедините похожие члены 92 = 8w − 4 Решите для w.96 = 8w12 = w

Используйте 3w − 2, чтобы найти длину.

l = 3w − 2 = 3 (12) −2 = 36−2 = 34

Для проверки убедитесь, что периметр 92 метра.

P = 2l + 2w = 2 (34) +2 (12) = 68 + 24 = 92

Ответ: Размер прямоугольника 12 на 34 метра.

Пример 14

При годовой процентной ставке 438%, сколько времени потребуется 2500 долларов, чтобы получить 437 долларов.50 простых процентов?

Решение:

Пусть t представляет время, необходимое для заработка 437,50 долларов при 438%. Организуйте информацию, необходимую для использования формулы для простых процентов, I = prt.

Заданные проценты за период:

I = 437,50 долларов США

Выдан принципал:

p = 2 500 долларов США

Задана оценка:

r = 438% = 4.375% = 0,04375

Затем подставьте все известные величины в формулу и найдите единственное неизвестное, t .

I = prt437,50 = 2500 (0,04375) t437,50 = 109,375t437,50109,375 = 109,375t109,3754 = t

Ответ: Чтобы заработать 437,50 долларов в виде простых процентов, требуется 4 года для 2500 долларов, вложенных под 438%.

Пример 15

Сьюзен вложила свои сбережения в размере 12 500 долларов в два счета, на которые были начислены простые проценты.Ее счет во взаимном фонде в прошлом году заработал 7%, а ее компакт-диск — 4,5%. Если ее общая сумма процентов за год составляла 670 долларов, сколько было на каждом счете?

Решение:

Связь между двумя неизвестными такова, что они составляют 12 500 долларов. Когда используется общая сумма, общий метод, используемый для исключения двух переменных, состоит в том, чтобы представить вторую неизвестную как разность общей суммы и первой неизвестной.

Пусть x представляет сумму, вложенную в паевой инвестиционный фонд.

Пусть 12,500 — x представляют оставшуюся сумму, вложенную в компакт-диск.

Организуйте данные.

Процентные доходы паевого инвестиционного фонда:

I = prt = x⋅0,07⋅1 = 0,07x

Проценты, полученные на CD:

I = prt = (12,500 − x) ⋅0.045⋅1 = 0,045 (12,500 − x)

Итого проценты:

$ 670

Общий процент — это сумма процентов, полученных с каждого счета.

проценты паевых инвестиционных фондов + проценты по CD = общая процентная ставка 0,07x + 0,045 (12,500 − x) = 670

Это уравнение моделирует проблему с одной переменной.Решите для x .

0,07x + 0,045 (12,500 − x) = 6700,07x + 562,5−0,045x = 6700,025x + 562,5 = 6700,025x = 107,5x = 107,50,025x = 4,300

Используйте 12,500 − x, чтобы найти сумму на компакт-диске.

12 500 − x = 12 500−4 300 = 8 200

Ответ: Сьюзен инвестировала 4300 долларов под 7% в паевой инвестиционный фонд и 8 200 долларов под 4,5% в компакт-диск.

Основные выводы

  • Решение общих линейных уравнений включает выделение переменной с коэффициентом 1 по одну сторону от знака равенства.Для этого сначала используйте соответствующее свойство равенства сложения или вычитания, чтобы изолировать член переменной с одной стороны от знака равенства. Затем изолируйте переменную, используя свойство равенства умножения или деления. Наконец, убедитесь, что ваше решение решает исходное уравнение.
  • Если решение линейного уравнения приводит к истинному утверждению типа 0 = 0, тогда уравнение является тождественным, а набор решений состоит из всех действительных чисел,,.
  • Если решение линейного уравнения приводит к ложному утверждению, например 0 = 5, то уравнение является противоречащим и решения не существует, Ø.
  • Очистите дроби, умножив обе части уравнения на наименьшее общее кратное всех знаменателей. Распределите и умножьте все члены на ЖК-дисплее, чтобы получить эквивалентное уравнение с целыми коэффициентами.
  • Упростите процесс решения реальных проблем, создав математические модели, описывающие взаимосвязь между неизвестными. Используйте алгебру, чтобы решить полученные уравнения.

Тематические упражнения

    Часть A: Решение основных линейных уравнений

      Определите, является ли данное значение решением.

    1. −5x + 4 = −1; х = -1

    2. 4x − 3 = −7; х = -1

    3. −2y + 7 = 12; у = -52

    4. 3a − 6 = 18 − a; а = −3

    5. 5 (2t − 1) = 2 − t; т = 2

      Решить.

    1. 23x + 12 = 1

    2. 38x + 54 = 32

    3. 1−3y5 = 2

    4. 2−5y6 = −8

    Часть B: Решение линейных уравнений

      Решить.

    1. 1.2x − 0,5−2,6x = 2−2,4x

    2. 1,59−3,87x = 3,48−4,1x − 0,51

    3. 4 (4a − 1) = 5 (a − 3) +2 (a − 2)

    4. 6 (x − 6) −3 (2x − 9) = — 9

    5. 54−12 (4y − 3) = 25 (y − 1)

    6. −2 (3x + 1) — (x − 3) = — 7x + 1

    7. Решить относительно F : C = 59 (F − 32)

    Часть C: Приложения

      Составьте алгебраическое уравнение и решите его.

      Количество проблем

    1. Если вычесть 3 из суммы числа и 10, получится 2. Найдите число.

    2. Сумма трех чисел и 12 равна 3. Найдите число.

    3. Трехкратная сумма числа и 6 равняется пятикратному количеству.Найдите номер.

    4. Дважды сумма числа и 4 равна трехкратной сумме числа и 1. Найдите число.

    5. Чем больше целое число, тем больше на единицу другого целого числа. Если сумма целых чисел равна 57, найдите целые числа.

    6. Большое целое число на 5 больше, чем удвоенное другое целое число. Если сумма целых чисел равна 83, найдите целые числа.

    7. Одно целое число на 3 меньше, чем удвоенное другое целое число. Найдите целые числа, если их сумма равна 135.

    8. Одно целое число на 10 меньше другого целого числа в 4 раза.Найдите целые числа, если их сумма равна 100.

    9. Сумма трех последовательных целых чисел равна 339. Найдите целые числа.

    10. Сумма четырех последовательных целых чисел равна 130. Найдите целые числа.

    11. Сумма трех последовательных четных целых чисел равна 174.Найдите целые числа.

    12. Сумма четырех последовательных четных целых чисел равна 116. Найдите целые числа.

    13. Сумма трех последовательных нечетных целых чисел равна 81. Найдите целые числа.

    14. Сумма четырех последовательных нечетных целых чисел равна 176.Найдите целые числа.

      Проблемы геометрии

    1. Длина прямоугольника на 5 сантиметров меньше его ширины в два раза. Если периметр 134 сантиметра, найдите длину и ширину.

    2. Длина прямоугольника на 4 сантиметра больше его ширины более чем в 3 раза.Если периметр 64 сантиметра, найдите длину и ширину.

    3. Ширина прямоугольника составляет половину его длины. Если периметр составляет 36 дюймов, найдите размеры прямоугольника.

    4. Ширина прямоугольника на 4 дюйма меньше его длины.Если периметр составляет 72 дюйма, найдите размеры прямоугольника.

    5. Периметр квадрата 48 дюймов. Найдите длину каждой стороны.

    6. Периметр равностороннего треугольника 96 дюймов. Найдите длину каждой стороны.

    7. Длина окружности составляет 80π единиц. Найдите радиус.

    8. Окружность круга составляет 25 сантиметров. Найдите радиус, округленный до сотых.

      Простые задачи по интересам

    1. На сколько лет нужно инвестировать 1000 долларов под 512%, чтобы заработать 165 долларов в виде простых процентов?

    2. На сколько лет нужно вложить 20 000 долларов под 614%, чтобы заработать 3 125 долларов в виде простых процентов?

    3. По какой годовой процентной ставке необходимо инвестировать 6500 долларов на 2 года, чтобы получить 1 040 долларов в виде простых процентов?

    4. По какой годовой процентной ставке необходимо инвестировать 5 750 долларов на 1 год, чтобы получить 333 доллара.50 простых процентов?

    5. Если простой процент, полученный за 5 лет, составлял 1860 долларов, а годовая процентная ставка составляла 6%, каков был основной процент?

    6. Если простой процент, полученный за 2 года, составлял 543,75 доллара, а годовая процентная ставка составляла 334%, какова была основная сумма?

    7. Сколько лет потребуется 600 долларов, чтобы удвоить простой процент по ставке 5% годовых? (Подсказка: для удвоения инвестиции должны приносить 600 долларов в виде простых процентов.)

    8. Сколько лет потребуется 10 000 долларов, чтобы удвоить простой процент по ставке 5% годовых? (Подсказка: для удвоения инвестиции должны приносить 10 000 долларов в виде простых процентов.)

    9. Джим вложил 4200 долларов в два счета. Один аккаунт приносит 3% простых процентов, а другой — 6%.Если через год процентная ставка составила 159 долларов, сколько он вложил в каждую учетную запись?

    10. Джейн вложила свои сбережения в размере 6500 долларов в два счета. Часть ее находится на компакт-диске под 5% годовых, а остальная часть — на сберегательном счете, на котором годовая процентная ставка составляет 4%. Если простой процент, полученный с обоих счетов, составляет 303 доллара в год, то сколько у нее есть на каждом счете?

    11. Хосе поместил прошлогодний бонус в размере 8 400 долларов на два счета.Он вложил часть в компакт-диск с 2,5% годовых, а остальную часть — в фонд денежного рынка с годовой процентной ставкой 1,5%. Его общая сумма процентов за год составила 198 долларов. Сколько он вложил в каждую учетную запись?

    12. Мэри вложила свои сбережения в размере 3300 долларов в два счета. Ее счет во взаимном фонде в прошлом году заработал 6,2%, а ее компакт-диск — 2,4%. Если бы ее общий процент за год составлял 124 доллара.80, сколько было на каждом счете?

    13. Алиса вкладывает деньги в два счета, один с 3% годовых, а другой с 5% годовых. Она вкладывает в счет с более высокой доходностью в 3 раза больше, чем в счет с более низкой доходностью. Если ее общий процент за год составляет 126 долларов, сколько она вложила в каждую учетную запись?

    14. Джеймс вложил наследство в два отдельных банка.Один банк предложил 512% годовых, другой — 614%. Он вложил в более доходный банковский счет вдвое больше, чем в другой. Если его общая простая процентная ставка за 1 год составляла 5760 долларов, то какова была сумма его наследства?

      Проблемы с равномерным движением

    1. Если Джиму требуется 114 часов, чтобы проехать 40 миль до работы, то какова средняя скорость Джима?

    2. Джил потребовалось 312 часов, чтобы проехать 189 миль до дома из колледжа.Какая у нее была средняя скорость?

    3. С какой скоростью должен двигаться Джим, если он хочет проехать 176 миль за 234 часа?

    4. Джеймс и Мартин смогли проехать 1140 миль от Лос-Анджелеса до Сиэтла. Если общая поездка заняла 19 часов, то какова была их средняя скорость?

    Часть D: Обсуждение

    1. Что считается основным делом алгебры? Объяснять.

    2. Каково происхождение слова алгебра ?

    3. Создайте собственное лицо или противоречие и поделитесь им на доске обсуждений. Предложите решение и объясните, как вы его нашли.

    4. Разместите в этом разделе что-нибудь, что вы нашли особенно полезным или интересным.Объяснить, почему.

    5. Выполните поиск в Интернете по запросу «решение линейных уравнений». Поделитесь ссылкой на веб-сайт или видеоруководством, которое, по вашему мнению, будет полезным.

Ответы

  1. ш = P − 2l2

  2. t = Dr

  3. б = 2Ач

  4. а = 2Ah − b

  5. F = 95C + 32

  1. Ширина: 24 сантиметра; длина: 43 см

  2. Ширина: 6 дюймов; длина: 12 дюймов

  3. Он инвестировал 3100 долларов под 3% и 1100 долларов под 6%.

  4. Хосе инвестировал 7200 долларов в CD и 1200 долларов в фонд денежного рынка.

  5. Алиса инвестировала 700 долларов под 3% и 2100 долларов под 5%.

Решение уравнений с одной переменной: TEAS || Зарегистрированныйorg

Основные термины и терминология, относящиеся к решению уравнений с одной переменной

  • Алгебра: Раздел математики, в котором есть хотя бы одно неизвестное число или переменная.
  • Переменная: неизвестное число в алгебраическом уравнении
  • Обратные математические вычисления: Противоположные вычисления
  • Обратный математический расчет для сложения: Вычитание
  • Обратный математический расчет для вычитания: Сложение
  • Обратный математический расчет для умножения: деление
  • Обратный математический расчет для деления: Умножение

Алгебраические уравнения

Обозначение алгебраических выражений:

1 — степень (показатель степени)
2 — коэффициент
3 — член
4 — оператор
5 — постоянный член
x y c — переменные / константы

Проще говоря, алгебра — это раздел математики, в котором есть по крайней мере одно неизвестное число или переменная.

Переменная в алгебре — это неизвестное число в алгебраическом уравнении. Переменные выражаются любой буквой, но x — это наиболее часто используемая буква переменной, которая используется в алгебре с одной неизвестной переменной, а любые другие буквы используются, когда в алгебраическом уравнении есть более одного неизвестного числа или переменной, например показано на картинке выше. Наиболее часто используемая буква для второй переменной — y, однако, как указано выше, может использоваться любая буква.

В этом разделе вы узнаете, как решать алгебраические уравнения только с одним неизвестным числом или переменной в алгебраическом уравнении, поэтому в этом разделе мы будем использовать только x в качестве неизвестного числа или переменной в алгебраическом уравнении, хотя любая буква может быть используется, как указано выше.

Решение алгебраических уравнений

Процедура решения алгебраических уравнений следующая:

  1. Поместите все неизвестные переменные (x) в левую часть знака равенства (=)
  2. Поместите все константы или известные числа справа от знака равенства (=)
  3. Выполняйте обратные математические вычисления для всех неизвестных или x, пока только 1 x или x не окажется слева от знака равенства (=)

Обратные математические вычисления — это противоположные вычисления.

Например:

  • Обратным математическим вычислением для сложения является вычитание.
  • Обратным математическим вычислением для вычитания является сложение.
  • Обратным математическим вычислением умножения является деление.
  • Обратным математическим вычислением деления является умножение.

Обратные математические вычисления важны и необходимы в алгебре, но они также полезны при проверке ваших ответов на вычисления сложения, вычитания, умножения и деления.

Вот несколько примеров того, как вычисления сложения могут быть проверены и проверены с помощью обратного математического вычисления вычитания:

Расчет сложения:

24 + 56 = 80

Обратное вычисление вычитания:

80–24 = 56

80–56 = 24

Расчет сложения:

2 + 560 = 562

Обратное вычисление вычитания:

562 — 2 = 560

562–560 = 2

Вот несколько примеров того, как вычисления вычитания могут быть проверены и проверены с помощью обратного математического вычисления сложения:

Расчет вычитания:

35–27 = 8

Обратный расчет сложения:

8 + 27 = 35

Расчет вычитания:

55-20 = 35

Обратный расчет сложения:

35 + 25 = 55

Вот несколько примеров того, как вычисления умножения могут быть проверены и проверены с помощью обратного математического вычисления деления:

Расчет умножения:

35 х 50 = 1750

Обратное вычисление деления:

1750 ÷ 50 = 35

1750 ÷ 35 = 50

Расчет умножения:

2.7 х 8,4 = 22,68

Обратное вычисление деления:

22,68 ÷ 2,7 = 8,4

22,68 ÷ 8,4 = 2,7

Вот несколько примеров того, как вычисления деления могут быть проверены и проверены с помощью обратного математического вычисления умножения:

Расчет отдела:

722 ÷ 4 = 180,5

Обратное вычисление умножения:

180,5 x 4 = 722

Расчет отдела:

82.6 ÷ 4 = 20,65

Обратное вычисление умножения:

20,65 x 4 = 82,6

Построение и решение алгебраических уравнений

Вот несколько примеров построения алгебраических уравнений с x в левой части и константами в правой части алгебраического уравнения:

Пример 1: Постановка и обработка алгебраических уравнений

11 х + 2 = 46

  • Установить с x слева и константой справа от знака =:

11 х + 2 = 46

Пример 2: Постановка и обработка алгебраических уравнений

х + 30 = 110

  • Установить с x слева и константой справа от знака =:

х + 30 = 110

Пример 3: Постановка и обработка алгебраических уравнений

14 — 3 = 3 х + 2

  • Установите с x слева и константой справа от знака =:

3 х + 2 = 14 — 3

Пример 4: Установка и обработка алгебраических уравнений

46 -2 = 11 х

  • Установить с x слева и константой справа от знака =:

11 х = 46 — 2

Пример 5: Установка и обработка алгебраических уравнений

33.3 х + 7 = 276

  • Установить с x слева и константой справа от знака =:

33,3 х + 7 = 276

Теперь, когда уравнения составлены, как указано выше, пришло время решить уравнение и узнать точное значение неизвестного x. Процедура этого показана ниже для каждого из пяти приведенных выше уравнений.

Пример 1: Решение алгебраического уравнения

  1. 11 х + 2 = 46
  2. 11 х = 46 — 2
  3. 11 х = 44
  4. х = 44 ÷ 11
  5. х = 4

Ответ: 4

Пример 2: Решение алгебраического уравнения

  • х + 30 = 110
  • х = 110 — 30
  • х = 80

Ответ: 80

Пример 3: Решение алгебраического уравнения

  • 3 x + 2 = 14 — 3
  • 3 х = 14 — 3 — 2
  • 3 х = 9
  • х = 9 ÷ 3
  • х = 3

Ответ: 3

Пример 4: Решение алгебраического уравнения

  • 11 x = 46 — 2
  • 11 х = 44
  • х = 44 ÷ 11
  • х = 4

Ответ: 4

Пример 5: Решение алгебраического уравнения

  • 33.3 х + 7 = 276 900 10
  • 33,3 х = 276-7
  • 33,3 х = 269
  • х = 269 ÷ 33,3
  • x = 8,08 с округлением до сотых

Ответ: 8.08

Вот несколько примеров выполнения обратных или противоположных вычислений для неизвестных (x) в алгебраическом уравнении:

Пример 1

Алгебраическое уравнение: 2 x + 7 = 19

Решение уравнения с обратными вычислениями:

Выполните обратный расчет.Вычтем 7 из обеих частей знака равенства (=)

  • 2 x + 7-7 = 19-7
  • 2 х = 12
  • Разделите обе части знака равенства (=) на 2.

Ответ: x = 6

Чтобы проверить этот ответ, подставьте 6 вместо каждого x в исходном алгебраическом уравнении, как показано ниже:

  • 2 (6) + 7 = 19
  • 12 + 7 = 19
  • 19 = 19

Когда числа (19) одинаковы по обе стороны от знака равенства, вычисление x как 6 является правильным.Когда числа разные для каждой стороны знака равенства, расчет неверен и неверен.

Пример 2

Алгебраическое уравнение: 2 x — 7 = 19

Решение уравнения с обратными вычислениями:

Выполните обратный расчет. Добавьте 7 к обеим сторонам знака равенства (=)

  • 2 х — 7 + 7 = 19 + 7
  • 2 х = 26
  • Разделите обе части знака равенства (=) на 2.

Ответ: x = 13

Чтобы проверить этот ответ, подставьте 13 вместо каждого x в исходном алгебраическом уравнении, как показано ниже:

  • 2 х — 7 = 19
  • 2 (13) — 7 = 19
  • 26-7 = 19
  • 19 = 19

Когда числа (19) одинаковы по обе стороны от знака равенства, вычисление x как 13 является правильным.Когда числа разные для каждой стороны знака равенства, расчет неверен и неверен.

Упрощенные алгебраические уравнения

Помимо решения алгебраических уравнений, как вы узнали выше, вы также можете увидеть другие типы алгебраических вычислений на вашем TEAS. Например, вы можете получить упрощенные вычисления алгебры и решение для x с помощью задачи со словом.

Вас могут спросить, каково решение или ответ для вычисления алгебры, такого как приведенные ниже.

Пример 1

Каков ответ на 2 x + 6 x — 2, когда x равно 4?

  • 2 (4) + 6 (4) — 2
  • 8 + 24-2 = 30

Ответ: 30

Пример 2

Каков ответ на 3 x + 2 x + 2, когда x равно 8?

  • 3 (8) + 2 (8) + 2
  • 24 + 16 + 2 = 42

Ответ: 42

Пример 3

Каков ответ на 3 2 x + 2 x + 2, когда x равен 5?

  • 9 (5) + 2 (5) + 2
  • 45 + 10 + 2 = 57

Ответ: 57

Пример 4

Каков ответ на 2 2 x — 2 x / 2, когда x равно 8?

  • 4 (8) — 2 (8) / 2
  • 32-16 ÷ 2
  • 16 ÷ 2 = 8

Ответ: 8

Алгебраические задачи со словами

У вас могут быть проблемы со словами на экзамене TEAS, которые необходимо решить и вычислить с помощью алгебры.

Самый простой способ определить, нужно ли решать проблему со словами и вычислять ее с помощью алгебры, — это задать вопрос: «Есть ли какие-нибудь неизвестные в этой задаче со словами?» Если ответ на этот вопрос утвердительный, то эту словесную задачу необходимо решить и вычислить с помощью алгебры. Если ответ на этот вопрос отрицательный, то эту словесную задачу НЕ нужно решать и вычислять с помощью алгебры.

Ниже приведены некоторые алгебраические задачи со словами:

Задача с алгебраическими словами № 1:

Сколько вам нужно было бы положить на свой сберегательный счет, чтобы утроить свои деньги на этом счете, если в настоящее время у вас есть 657 долларов.

Ответ: Вам нужно будет положить 1971 доллар на свой сберегательный счет.

Задача с алгебраическими словами № 2:

Что на 5 больше, чем на 2 дюжины?

  • х = 2 (12) + 5
  • х = 24 + 5
  • х = 29

Ответ: 29 — это 5 больше, чем 2 десятка. Их 12 в одной дюжине.

Алгебраическая задача со словами № 3:

Что составляет 2/7 (одна седьмая) веса объекта, когда 3/7 (три седьмых) составляет 68 фунтов? (Этот расчет представляет собой двухэтапный расчет.Сначала вы должны вычислить, что такое 1/7, а затем вы должны вычислить, что составляет 2/7 веса объекта.

Узнаем, что такое 1/7:

  • 3/7: 68 = 2/7: x
  • 3/7 х = 68 х 2/7
  • 3 х = 68
  • x = 19,43 с округлением до сотых
  • 1/7 веса составляет 19,43 фунта, когда 3/7 веса составляет 68 фунтов
  • Следующий шаг — определить 2/7 веса, когда 1/7 равна 19,43.
  • х = 2 (19,43)
  • х = 38.86

Ответ: На 2/7 общего веса приходится 38,86 фунта, когда общий вес для 3/7 общего веса составляет 68 фунтов.

ЧИСЛА СВЯЗАННЫХ TEAS И СОДЕРЖАНИЕ АЛГЕБРЫ :

Alene Burke, RN, MSN

Alene Burke RN, MSN является национально признанным преподавателем сестринского дела. Она начала свою карьеру учителем начальной школы в Нью-Йорке, а затем поступила в общественный колледж Квинсборо, чтобы получить степень младшего специалиста по медсестринскому делу. Она работала дипломированной медсестрой в отделении интенсивной терапии местной общественной больницы, и в то время она решила стать преподавателем медсестер.Она получила степень бакалавра наук по медсестринскому делу в колледже Эксельсиор, который входит в состав Университета штата Нью-Йорк, и сразу после окончания школы она поступила в аспирантуру Университета Адельфи на Лонг-Айленде, штат Нью-Йорк. Она закончила Summa Cum Laude в Адельфи, получив двойную степень магистра в области сестринского образования и сестринского администрирования, и сразу же получила докторскую степень по сестринскому делу в том же университете. Она является автором сотен курсов для медицинских работников, включая медсестер, она работает медсестрой-консультантом в медицинских учреждениях и частных корпорациях, она также является утвержденным поставщиком непрерывного образования для медсестер и других дисциплин, а также была членом Американской ассоциации медсестер. Целевая группа ассоциации по компетентности и обучению членов медсестер.

Последние сообщения Alene Burke, RN, MSN (посмотреть все)

Решение линейных уравнений | Уравнения и неравенства

Упражнение 4.1

\ begin {align *}
2г — 3 & = 7 \\
2л & = 10 \\
y & = 5
\ end {выровнять *}

\ begin {align *}
2c & = c — 8 \\
c & = -8
\ end {выровнять *}

\ begin {align *}
3 & = 1 — 2c \\
2c & = 1 — (3) \\
2c & = -2 \\
c & = \ frac {-2} {2} \\
& = -1
\ end {align *}

\ begin {align *}
4b +5 & = -7 \\
4b & = -7 — (5) \\
4b & = -12 \\
b & = \ frac {-12} {4} \\
& = -3
\ end {align *}

\ begin {align *}
-3y & = 0 \\
у & = 0
\ end {выровнять *}

\ begin {align *}
16л + ​​4 & = -10 \\
16лет & = -14 \\
y & = — \ frac {14} {16} \\
& = — \ frac {7} {8}
\ end {выровнять *}

\ begin {align *}
12лет + 0 & = 144 \\
12лет & = 144 \\
y & = 12
\ end {выровнять *}

\ begin {align *}
7 + 5л & = 62 \\
5лет & = 55 \\
y & = 11
\ end {выровнять *}

\ (55 = 5x + \ frac {3} {4} \)

\ begin {align *}
55 & = 5x + \ frac {3} {4} \\
220 & = 20х + 3 \\
20x & = 217 \\
x & = \ frac {217} {20}
\ end {выровнять *}

\ begin {align *}
5х & = 2х + 45 \\
3x & = 45 \\
х & = 15
\ end {выровнять *}

\ begin {align *}
23х — 12 & = 6 + 3х \\
20x & = 18 \\
x & = \ frac {18} {20} \\
& = \ frac {9} {10}
\ end {выровнять *}

\ (12 — 6x + 34x = 2x — 24 — 64 \)

\ begin {align *}
12 — 6x + 34x & = 2x — 24 — 64 \\
12 + 28x & = 2x — 88 \\
26x & = -100 \\
x & = — \ frac {100} {26} \\
& = — \ frac {50} {13}
\ end {выровнять *}

\ (6x + 3x = 4-5 (2x — 3) \)

\ begin {align *}
6x + 3x & = 4-5 (2x — 3) \\
9x & = 4 — 10x + 15 \\
19x & = 19 \\
х & = 1
\ end {выровнять *}

\ begin {align *}
18 — 2р & = р + 9 \\
9 & = 3п \\
p & = 3
\ end {выровнять *}

\ (\ dfrac {4} {p} = \ dfrac {16} {24} \)

\ begin {align *}
\ frac {4} {p} & = \ frac {16} {24} \\
(4) (24) & = (16) (p) \\
16p & = 96 \\
p & = 6
\ end {выровнять *}

\ begin {align *}
— (- 16 — п) & = 13п — 1 \\
16 + п & = 13п — 1 \\
17 & = 12п \\
p & = \ frac {17} {12}
\ end {выровнять *}

\ begin {align *}
3f — 10 & = 10 \\
3f & = 20 \\
f & = \ frac {20} {3}
\ end {выровнять *}

\ begin {align *}
3f + 16 & = 4f — 10 \\
f & = 26
\ end {выровнять *}

\ (10f + 5 = -2f -3f + 80 \)

\ begin {align *}
10f + 5 & = -2f — 3f + 80 \\
10f + 5 & = -5f + 80 \\
15f & = 75 \\
f & = 5
\ end {выровнять *}

\ begin {align *}
8 (ф — 4) & = 5 (ф — 4) \\
8f — 32 & = 5f — 20 \\
3f & = 12 \\
f & = 4
\ end {выровнять *}

\ begin {align *}
6 & = 6 (f + 7) + 5f \\
6 & = 6f + 42 + 5f \\
-36 & = 11f \\
f & = — \ frac {36} {11}
\ end {выровнять *}

\ begin {align *}
-7x & = 8 (1 — х) \\
-7x & = 8 — 8x \\
х & = 8
\ end {выровнять *}

\ (5 — \ dfrac {7} {b} = \ dfrac {2 (b + 4)} {b} \)

\ begin {align *}
5 — \ frac {7} {b} & = \ frac {2 (b + 4)} {b} \\
\ frac {5b — 7} {b} & = \ frac {2b + 8} {b} \\
5b — 7 & = 2b + 8 \\
3b & = 15 \\
b & = 5
\ end {выровнять *}

\ (\ dfrac {x + 2} {4} — \ dfrac {x — 6} {3} = \ dfrac {1} {2} \)

\ begin {align *}
\ frac {x + 2} {4} — \ frac {x — 6} {3} & = \ frac {1} {2} \\
\ frac {3 (x + 2) — 4 (x — 6)} {12} & = \ frac {1} {2} \\
\ frac {3x + 6 — 4x + 24} {12} & = \ frac {1} {2} \\
(-x + 30) (2) & = 12 \\
-2x + 60 & = 12 \\
-2x & = -48 \\
х & = 24
\ end {выровнять *}

\ (1 = \ dfrac {3a — 4} {2a + 6} \)

Обратите внимание, что \ (a \ neq — -3 \)

\ begin {align *}
1 & = \ frac {3a — 4} {2a + 6} \\
2а + 6 & = 3а — 4 \\
а & = 10
\ end {выровнять *}

\ (\ dfrac {2-5a} {3} — 6 = \ dfrac {4a} {3} +2 — a \)

\ begin {align *}
\ frac {2-5a} {3} — 6 & = \ frac {4a} {3} +2 — a \\
\ frac {2-5a} {3} — \ frac {4a} {3} + a & = 8 \\
\ frac {2-5a — 4 a + 3a} {3} & = 8 \\
2 — 6а & = 24 \\
6а & = -22 \\
a & = — \ frac {22} {6}
\ end {выровнять *}

\ (2 — \ dfrac {4} {b + 5} = \ dfrac {3b} {b + 5} \)

Примечание \ (b \ neq -5 \)

\ begin {align *}
2 — \ frac {4} {b + 5} & = \ frac {3b} {b + 5} \\
2 & = \ frac {3b + 4} {b + 5} \\
2b + 10 & = 3b + 4 \\
b & = 6
\ end {выровнять *}

\ (3 — \ dfrac {y — 2} {4} = 4 \)

\ begin {align *}
3 — \ frac {y — 2} {4} & = 4 \\
— \ frac {y — 2} {4} & = 1 \\
-у + 2 & = 4 \\
y & = -2
\ end {выровнять *}

\ (\ text {1,5} x + \ text {3,125} = \ text {1,25} x \)

\ begin {align *}
\ text {1,5} x + \ text {3,125} & = \ text {1,25} x \\
\ text {1,5} x — \ text {1,25} x & = — \ text {3,125} \\
\ text {0,25} x & = — \ text {3,125} \\
х & = — \ текст {12,5}
\ end {выровнять *}

\ (\ текст {1,3} (\ текст {2,7} х + 1) = \ текст {4,1} — х \)

\ begin {align *}
\ text {1,3} (\ text {2,7} x + 1) & = \ text {4,1} — x \\
\ text {3,51} x + \ text {1,3} & = \ text {4,1} — x \\
\ text {4,51} x & = \ text {2,8} \\
x & = \ frac {\ text {2,8}} {\ text {4,51}} \\
& = \ frac {280} {451}
\ end {выровнять *}

\ (\ текст {6,5} х — \ текст {4,15} = 7 + \ текст {4,25} х \)

\ begin {align *}
\ text {6,5} x — \ text {4,15} & = 7 + \ text {4,25} x \\
\ text {2,25} x & = \ text {11,15} \\
x & = \ frac {\ text {11,15}} {\ text {2,25}} \\
& = \ frac {\ text {1 115}} {225} \\
& = \ frac {223} {45}
\ end {выровнять *}

\ (\ frac {1} {3} P + \ frac {1} {2} P — 10 = 0 \)

\ begin {align *}
\ frac {1} {3} P + \ frac {1} {2} P — 10 & = 0 \\
\ frac {2 + 3} {6} P & = 10 \\
5П & = 60 \\
P & = 12
\ end {выровнять *}

\ (1 \ frac {1} {4} (x — 1) — 1 \ frac {1} {2} (3x + 2) = 0 \)

\ begin {align *}
1 \ frac {1} {4} (x — 1) — 1 \ frac {1} {2} (3x + 2) & = 0 \\
\ frac {5} {4} x — \ frac {5} {4} — \ frac {3} {2} (3x) — \ frac {3} {2} (2) & = 0 \\
\ frac {5} {4} x — \ frac {5} {4} — \ frac {9} {2} x — \ frac {6} {2} & = 0 \\
\ frac {5 — 18} {4} x + \ frac {-5 — 12} {4} & = 0 \\
\ frac {-13} {4} x & = \ frac {17} {4} \\
-13x & = 17 \\
х & = — \ frac {17} {13}
\ end {выровнять *}

\ (\ frac {1} {5} (x- 1) = \ frac {1} {3} (x-2) + 3 \)

\ begin {align *}
\ frac {1} {5} (x- 1) & = \ frac {1} {3} (x-2) + 3 \\
\ frac {1} {5} x- \ frac {1} {5} & = \ frac {1} {3} x- \ frac {2} {3} + 3 \\
— \ frac {1} {5} + \ frac {2} {3} — 3 & = \ frac {2} {15} x \\
— \ frac {38} {15} & = \ frac {2} {15} x \\
х & = — \ frac {38} {2} \\
х & = -19
\ end {выровнять *}

\ (\ dfrac {5} {2a} + \ dfrac {1} {6a} — \ dfrac {3} {a} = 2 \)

\ begin {align *}
\ frac {5} {2a} + \ frac {1} {6a} — \ frac {3} {a} & = 2 \\
\ frac {5 (3) + 1-3 (6)} {6a} & = 2 \\
\ frac {15 + 1 — 18} {6a} & = 2 \\
\ frac {-2} {6a} & = 2 \\
-2 & = 12а \\
а & = — \ frac {1} {6}
\ end {выровнять *}

Системы линейных уравнений

Линейное уравнение — это уравнение для линии .

Линейное уравнение не всегда имеет вид y = 3,5 — 0,5x ,

Это также может быть как y = 0,5 (7 — x)

Или как y + 0,5x = 3,5

Или как y + 0,5x — 3,5 = 0 и более.

(Примечание: это одно и то же линейное уравнение!)

Система линейных уравнений — это когда у нас есть два или более линейных уравнения , работающих вместе.

Пример: Вот два линейных уравнения:

Вместе они представляют собой систему линейных уравнений.

Сможете ли вы сами определить значения x и y ? (Просто попробуйте, поиграйте с ними немного.)

Попробуем построить и решить реальный пример:

Пример: вы против лошади

Это гонка!

Вы можете бегать 0,2 км каждую минуту.

Лошадь может бежать 0,5 км каждую минуту. Но оседлать лошадь нужно за 6 минут.

Как далеко вы можете уйти, прежде чем лошадь вас поймает?

Мы можем составить два уравнения ( d = расстояние в км, t = время в минутах)

  • Вы бежите на 0.2 км каждую минуту, поэтому d = 0,2 т
  • Лошадь бежит со скоростью 0,5 км в минуту, но мы берем на ее время 6: d = 0,5 (t − 6)

Итак, у нас есть система уравнений (это линейных ):

Решаем на графике:

Вы видите, как лошадь начинает через 6 минут, а потом бежит быстрее?

Кажется, тебя поймают через 10 минут … Тебе всего 2 км.

В следующий раз беги быстрее.

Итак, теперь вы знаете, что такое система линейных уравнений.

Давайте продолжим узнавать о них больше ….

Решение

Существует множество способов решения линейных уравнений!

Давайте посмотрим на другой пример:

Пример: Решите эти два уравнения:

На этом графике показаны два уравнения:

Наша задача — найти место пересечения двух линий.

Ну, мы видим, где они пересекаются, так что это уже решено графически.

А теперь давайте решим это с помощью алгебры!

Хммм … как это решить? Способов может быть много! В этом случае оба уравнения имеют «y», поэтому давайте попробуем вычесть все второе уравнение из первого:

x + y — (−3x + y) = 6 — 2

А теперь упростим:

х + у + 3х — у = 6-2

4x = 4

х = 1

Итак, теперь мы знаем, что линии пересекаются в точке x = 1 .

И мы можем найти совпадающее значение y , используя любое из двух исходных уравнений (потому что мы знаем, что они имеют одинаковое значение при x = 1). Воспользуемся первым (второй можете попробовать сами):

х + у = 6

1 + у = 6

г = 5

И решение:

x = 1 и y = 5

И график показывает, что мы правы!

Линейные уравнения

В линейных уравнениях допускаются только простые переменные. Нет x 2 , y 3 , √x и т. Д. :

Линейное против нелинейного

Размеры

Линейное уравнение может быть в двух измерениях …
(например, x и y )
… или в 3-х измерениях …
(делает самолет)
… или 4 размера …
… или больше!

Общие переменные

Чтобы уравнения «работали вместе», они разделяют одну или несколько переменных:

Система уравнений содержит два или более уравнений в одну или несколько переменных

Множество переменных

Таким образом, Система уравнений может иметь много уравнений и много переменных.

Пример: 3 уравнения с 3 переменными

2x + y 2z = 3
x y z = 0
x + y + 3z = 12

Может быть любая комбинация:

  • 2 уравнения с 3 переменными,
  • 6 уравнений с 4 переменными,
  • 9000 уравнений в 567 переменных,
  • и др.

Решения

Когда количество уравнений равно , то же , что и количество переменных, , вероятно, будет решением. Не гарантировано, но вероятно.

На самом деле есть только три возможных случая:

  • Нет раствор
  • Одно решение
  • Бесконечно много решений

Когда нет решения , уравнения называются «несовместимыми» .

Одно или бесконечно много решений называются «согласованными»

Вот диаграмма для 2 уравнения с 2 переменными :

Независимый

«Независимый» означает, что каждое уравнение дает новую информацию.
В противном случае это «Зависимые» .

Также называется «линейная независимость» и «линейная зависимость»

Пример:

Эти уравнения — «Зависимые» , потому что на самом деле это то же уравнение , только умноженное на 2.

Итак, второе уравнение не дало новой информации .

Где верны уравнения

Уловка состоит в том, чтобы найти, где все уравнения являются истинными одновременно .

Верно? Что это значит?

Пример: вы против лошади

Линия «ты» истинна по всей ее длине (но больше нигде).

В любом месте этой строки d равно 0.2т

  • при t = 5 и d = 1 уравнение истинно (d = 0,2t? Да, поскольку 1 = 0,2 × 5 истинно)
  • при t = 5 и d = 3, уравнение неверно (Является ли d = 0,2t? Нет, поскольку 3 = 0,2 × 5 неверно )

Точно так же линия «лошади» истинна по всей своей длине (но больше нигде).

Но только в точке, где они пересекают (при t = 10, d = 2), они оба истинны .

Значит, они должны быть правдой одновременно

… поэтому некоторые люди называют их «Одновременные линейные уравнения»

Решить с помощью алгебры

Для их решения принято использовать алгебру.

Вот пример «Лошади», решенный с помощью алгебры:

Пример: вы против лошади

Система уравнений:

В этом случае кажется, что проще всего установить их равными друг другу:

d = 0.2т = 0,5 (т − 6)

Начать с : 0,2t = 0,5 (t — 6)

Расширить 0,5 (t − 6) : 0,2t = 0,5t — 3

Вычтем 0,5t с обеих сторон: −0,3t = −3

Разделим обе части на −0,3 : t = −3 / −0,3 = 10 минут

Теперь мы знаем , когда тебя поймают!

Зная t , можно вычислить d : d = 0,2t = 0,2 × 10 = 2 км

И наше решение:

t = 10 минут и d = 2 км

Алгебра и графики

Зачем использовать алгебру, если графики настолько просты? Потому что:

Более 2 переменных не могут быть решены с помощью простого графика.

Итак, алгебра приходит на помощь двумя популярными методами:

  • Решение заменой
  • Решение методом исключения

Мы увидим каждую с примерами по 2 переменным и 3 переменным. Вот и …

Решение заменой

Это шаги:

  • Напишите одно из уравнений в стиле «переменная = …»
  • Заменить (т.е. заменить) эту переменную в другое уравнение (а).
  • Решите другое уравнение (а)
  • (при необходимости повторить)

Вот пример с 2 уравнениями с 2 переменными :

Пример:

Мы можем начать с любого уравнения и любой переменной .

Воспользуемся вторым уравнением и переменной «y» (это выглядит как простейшее уравнение).

Напишите одно из уравнений в стиле «переменная =»… «:

Мы можем вычесть x из обеих частей x + y = 8, чтобы получить y = 8 — x . Теперь наши уравнения выглядят так:

Теперь замените «y» на «8 — x» в другом уравнении:

  • 3x + 2 (8 — x) = 19
  • у = 8 — х

Решите, используя обычные методы алгебры:

Развернуть 2 (8 − x) :

  • 3x + 16 — 2x = 19
  • у = 8 — х

Тогда 3x − 2x = x :

И на последок 19-16 = 3

Теперь мы знаем, что такое x , мы можем поместить его в уравнение y = 8 — x :

И ответ:

х = 3
у = 5

Примечание: поскольку — это решение, уравнения «непротиворечивы»

Проверка: почему бы вам не проверить, работают ли x = 3 и y = 5 в обоих уравнениях?

Решение подстановкой: 3 уравнения с 3 переменными

ОК! Давайте перейдем к более длинному примеру : 3 уравнения с 3 переменными .

Это несложно, сделать … это займет у много времени !

Пример:

  • х + г = 6
  • г — 3у = 7
  • 2x + y + 3z = 15

Мы должны аккуратно выровнять переменные, иначе мы потеряем из виду, что делаем:

x + z = 6
3 года + z = 7
2x + y + 3z = 15

WeI может начать с любого уравнения и любой переменной.Воспользуемся первым уравнением и переменной «x».

Напишите одно из уравнений в стиле «переменная = …»:

x = 6 — z
3 года + z = 7
2x + y + 3z = 15

Теперь замените «x» на «6 — z» в других уравнениях:

(К счастью, есть только одно уравнение с x в нем)

х = 6 — z
3 года + z = 7
2 (6-z) + y + 3z = 15

Решите, используя обычные методы алгебры:

2 (6 − z) + y + 3z = 15 упрощается до y + z = 3 :

x = 6 — z
3 года + z = 7
y + z = 3

Хорошо.Мы добились некоторого прогресса, но пока не достигли этого.

Теперь повторите процесс , но только для последних 2 уравнений.

Напишите одно из уравнений в стиле «переменная = …»:

Выберем последнее уравнение и переменную z:

x = 6 — z
3 года + z = 7
z = 3 — х лет

Теперь замените «z» на «3 — y» в другом уравнении:

x = 6 — z
3 года + 3 — х лет = 7
z = 3-й год

Решите, используя обычные методы алгебры:

−3y + (3 − y) = 7 упрощается до −4y = 4 , или другими словами y = −1

x = 6 — z
и = -1
z = 3-й год

Почти готово!

Зная, что y = −1 , мы можем вычислить, что z = 3 − y = 4 :

x = 6 — z
y = -1
z = 4

И зная, что z = 4 , мы можем вычислить, что x = 6 − z = 2 :

x = 2
y = -1
z = 4

И ответ:

х = 2
у = -1
г = 4

Проверка: проверьте сами.

Мы можем использовать этот метод для 4 или более уравнений и переменных … просто повторяйте одни и те же шаги снова и снова, пока она не будет решена.

Заключение: Замена работает хорошо, но требует много времени.

Решение методом исключения

Уничтожение может быть быстрее … но должно быть аккуратным.

«Исключить» означает удалить : этот метод работает путем удаления переменных до тех пор, пока не останется только одна.

По идее, у нас можно смело :

  • умножить уравнение на константу (кроме нуля),
  • прибавить (или вычесть) уравнение к другому уравнению

Как в этих примерах:

ПОЧЕМУ мы можем складывать уравнения друг в друга?

Представьте себе два действительно простых уравнения:

х — 5 = 3
5 = 5

Мы можем добавить «5 = 5» к «x — 5 = 3»:

х — 5 + 5 = 3 + 5
х = 8

Попробуйте сами, но используйте 5 = 3 + 2 в качестве второго уравнения

Он по-прежнему будет работать нормально, потому что обе стороны равны (для этого стоит знак =!)

Мы также можем поменять местами уравнения, чтобы первое могло стать вторым и т. Д., Если это поможет.

Хорошо, время для полного примера. Давайте использовать 2 уравнения с 2 переменными , пример из предыдущего:

Пример:

Очень важно, чтобы все было в порядке:

3x + 2 года = 19
x + y = 8

Сейчас… наша цель — исключить переменную из уравнения.

Сначала мы видим, что есть «2y» и «y», так что давайте поработаем над этим.

Умножьте второе уравнение на 2:

.

3x + 2 года = 19
2 x + 2 y = 16

Вычтем второе уравнение из первого уравнения:

x = 3
2x + 2 года = 16

Ура! Теперь мы знаем, что такое x!

Затем мы видим, что во втором уравнении есть «2x», поэтому давайте уменьшим его вдвое, а затем вычтем «x»:

Умножьте второе уравнение на ½ (т.е.е. разделить на 2):

x = 3
x + и = 8

Вычтем первое уравнение из второго уравнения:

x = 3
и = 5

Готово!

И ответ:

x = 3 и y = 5

А вот график:

Синяя линия — это где 3x + 2y = 19 истинно

Красная линия — это где x + y = 8 истинно

При x = 3, y = 5 (где линии пересекаются) они равны , оба истинны. Это и есть ответ.

Вот еще один пример:

Пример:

  • 2х — у = 4
  • 6x — 3y = 3

Разложите аккуратно:

2x y = 4
6x 3 года = 3

Умножьте первое уравнение на 3:

6x 3 года = 12
6x 3 года = 3

Вычтем второе уравнение из первого уравнения:

0 0 = 9
6x 3 года = 3

0-0 = 9 ???

Что здесь происходит?

Все просто, решения нет.

На самом деле это параллельные линии:

И на последок:

Пример:

  • 2х — у = 4
  • 6x — 3y = 12

Аккуратно:

2x y = 4
6x 3 года = 12

Умножьте первое уравнение на 3:

6x 3 года = 12
6x 3 года = 12

Вычтем второе уравнение из первого уравнения:

0 0 = 0
6x 3 года = 3

0 — 0 = 0

Ну, это на самом деле ИСТИНА! Ноль действительно равен нулю…

… это потому, что на самом деле это одно и то же уравнение …

… так что существует бесконечное количество решений

Это та же строка:

Итак, теперь мы рассмотрели пример каждого из трех возможных случаев:

  • Нет раствор
  • Одно решение
  • Бесконечно много решений

Решение методом исключения: 3 уравнения с 3 переменными

Прежде чем мы начнем со следующего примера, давайте рассмотрим улучшенный способ решения задач.

Следуйте этому методу, и мы с меньшей вероятностью ошибемся.

Прежде всего удалите переменные в порядке :

  • Сначала удалите x с (из уравнений 2 и 3, по порядку)
  • , затем исключите y (из уравнения 3)

Вот как мы их устраняем:

У нас есть «форма треугольника»:

Теперь начните снизу и вернитесь к (так называемая «обратная подстановка»)
(введите z , чтобы найти y , затем z и y , чтобы найти x ):

И решаемся:

ТАКЖЕ, мы обнаружим, что легче выполнить расчетов в уме или на бумаге для заметок, чем всегда работать в рамках системы уравнений:

Пример:

  • х + у + г = 6
  • 2y + 5z = −4
  • 2x + 5y — z = 27

Аккуратно написано:

x + y + z = 6
2 года + 5z = −4
2x + 5лет z = 27

Сначала удалите x из 2-го и 3-го уравнения.

Во втором уравнении нет x … переходите к третьему уравнению:

Вычтите 2 раза 1-е уравнение из 3-го уравнения (просто сделайте это в уме или на бумаге для заметок):

И получаем:

x + y + z = 6
2 года + 5z = −4
3 года 3z = 15

Затем удалите y из 3-го уравнения.

Мы, , могли бы вычесть 1½ раза 2-е уравнение из 3-го уравнения (потому что 1½ раза 2 равно 3) …

… но мы можем избежать дробей , если мы:

  • умножьте 3-е уравнение на 2 и
  • умножьте 2-е уравнение на 3

и , затем выполняют вычитание … вот так:

И в итоге получаем:

x + y + z = 6
2 года + 5z = −4
z = -2

Теперь у нас есть «треугольная форма»!

Теперь вернемся снова вверх «с ​​обратной заменой»:

Мы знаем z , поэтому 2y + 5z = −4 становится 2y − 10 = −4 , затем 2y = 6 , поэтому y = 3 :

x + y + z = 6
и = 3
z = −2

Тогда x + y + z = 6 становится x + 3−2 = 6 , поэтому x = 6−3 + 2 = 5

x = 5
y = 3
z = −2

И ответ:

x = 5
y = 3
z = −2

Проверка: проверьте сами.

Общий совет

Как только вы привыкнете к методу исключения, он станет проще, чем замена, потому что вы просто выполняете шаги, и ответы появляются.

Но иногда замена может дать более быстрый результат.

  • Замена часто проще для небольших случаев (например, 2 уравнения, а иногда и 3 уравнения)
  • Устранение проще для больших ящиков

И всегда полезно сначала просмотреть уравнения, чтобы увидеть, есть ли простой ярлык… так что опыт помогает.

Обучение линейным уравнениям в математике

Для многих учеников 8-х классов и выше числа и формы, которые они узнали, действительно начинают сходиться воедино, когда они составляют и решают линейные уравнения. Эта тема объединяет идеи об алгебре, геометрии и функциях, и многим детям — и взрослым может быть сложно осмыслить. В этой статье объясняется, что такое линейное уравнение, и рассматриваются различные примеры.Затем он предлагает учащимся идеи уроков по введению и развитию концепции линейных уравнений с одной переменной.

Что такое линейное уравнение?

Как и любое другое уравнение, линейное уравнение состоит из двух равных друг другу выражений. Есть некоторые ключевые особенности, общие для всех линейных уравнений:

  1. Линейное уравнение имеет только одну или две переменные.
  2. Никакая переменная в линейном уравнении не возводится в степень больше 1 или не используется в качестве знаменателя дроби.
  3. Когда вы находите пары значений, которые делают линейное уравнение истинным, и наносите эти пары на координатную сетку, все точки лежат на одной линии. График линейного уравнения представляет собой прямую линию.

Линейное уравнение с двумя переменными можно описать как линейное соотношение между x и y , то есть двумя переменными, в которых значение одной из них (обычно y ) зависит от значение другого (обычно x ).В этом случае x является независимой переменной, а y зависит от нее, поэтому y называется зависимой переменной.

Независимая переменная, помеченная как x , обычно отображается по горизонтальной оси. Большинство линейных уравнений — это функции. Другими словами, для каждого значения x существует только одно соответствующее значение y . Когда вы присваиваете значение независимой переменной x , вы можете вычислить значение зависимой переменной y .Затем вы можете нанести точки, названные каждой парой ( x , y ), на координатной сетке.

Описание линейных отношений

Студенты уже должны знать, что любые две точки определяют линию. Таким образом, для построения графика линейного уравнения на самом деле требуется всего лишь найти две пары значений и провести линию через точки, которые они описывают. Все остальные точки на линии предоставят значения x и y , которые удовлетворяют уравнению.

Графики линейных уравнений всегда представляют собой линии.Однако важно помнить, что не каждая точка на линии, которую описывает уравнение, обязательно будет решением проблемы, которую описывает уравнение. Например, проблема может не иметь смысла для отрицательных чисел (скажем, если независимая переменная — время) или очень больших чисел (скажем, чисел больше 100, если зависимая переменная — это оценка в классе).

Как выглядит линейное уравнение?

Пример 1: расстояние = скорость × время

В этом уравнении для любой заданной постоянной скорости соотношение между расстоянием и временем будет линейным.Однако расстояние обычно выражается положительным числом, поэтому на большинстве графиков этой взаимосвязи будут отображаться только точки в первом квадранте. Обратите внимание, что направление линии на графике ниже — снизу слева направо. Линии, идущие в этом направлении, имеют положительный наклон . Положительный наклон указывает, что значения на обеих осях увеличиваются слева направо.

линейных уравнений в одном листе переменных с ответами

Другими словами, уравнение вида ax + b = c, где a, b, c — константы, a ≠ 0 и ‘x’ — переменная, называется линейным уравнением в переменной … Линейные уравнения имеют вид форма ax + b = c.Обязательно соблюдайте все рабочие листы по математике для класса 7, подготовленные… Рабочим листом по линейным уравнениям. По мере развития этого навыка мы узнаем, что коэффициенты могут быть разными в одинаковых условиях. Часто требуется немного практики, чтобы преобразовать английское предложение в математическое. Уравнение, в котором наибольшая степень участвующих переменных равна 1, называется линейным уравнением. Это может быть базовое число или переменная. Прежде чем взглянуть на рабочий лист, если вы хотите узнать о решении линейных уравнений с одной переменной, нажмите здесь.Рабочий лист по математике для класса 8 CBSE — Линейные уравнения в одной переменной — Практические рабочие листы для студентов CBSE. Решение линейных уравнений. Вышеупомянутое линейное уравнение верно только в том случае, если x = 5, и, следовательно, данное линейное уравнение имеет только одно решение, то есть рабочие листы для решения и построения графиков линейных уравнений с ответами. Студенты должны бесплатно скачать и практиковать эти рабочие листы, чтобы получить больше оценок на экзаменах. Рабочий лист по математике CBSE для класса 8 — Линейные уравнения в одной переменной Давайте взглянем на некоторые концепции, которые обсуждаются в этой главе.Пример 1: Рассмотрим уравнение 7x — 35 = 0. Пример 2: Рассмотрим уравнение 9 (x — 1) — 35 = 8x + 37. Введение в линейное уравнение. Лист ответов. Эта страница состоит из рабочего листа субъективных вопросов для главы «Линейные уравнения в одной и двух переменных» класса 7 по математике с подробными решениями. Это то, на чем мы сосредоточимся здесь с некоторыми основными задачами счисления, проблемами геометрии и проблемами деталей. Ниже описаны несколько важных фраз, которые могут дать нам ключ к пониманию того, как установить Рабочий лист для класса 7 по математике для главы 6 ЛИНЕЙНЫХ УРАВНЕНИЙ В ОДНОЙ И ДВЕ ПЕРЕМЕННОЙ.Подготовлено преподавателями лучших школ CBSE Индии. При решении у нас есть 9x — 9 — 35 = 8x + 37 .. Линейное уравнение в одной сумме переменных — Отображение 8 основных рабочих листов, найденных для этой концепции .. Проблемы с линейными неравенствами. Обладает следующими навыками: приводить примеры линейных уравнений с одной переменной с одним решением, бесконечным множеством решений или без решения. Приведенные здесь решения RD Sharma для класса 7 содержат ответы на все вопросы, представленные в этих упражнениях. Собираем одинаковые термины с обеих сторон, передавая их, имеем А также, результат «-3 = -3» верен.CBSE Class 8 Linear Equations in One Variable Worksheet (1) Рабочие листы стали неотъемлемой частью системы образования. Рабочие листы очень важны для каждого ученика, чтобы практиковать свои концепции. При решении мы имеем 7x = 35 или x = 5. Например, значение -4yz 2 и yz 2/3 подобны членам. На последнем этапе решения данного уравнения переменной «x» больше нет. Глава 8 «Линейные уравнения в одной переменной» содержит четыре упражнения. Студенты, учителя и родители могут загрузить все учебные материалы CBSE и очень хорошо подготовленные рабочие листы с этого веб-сайта.x = 5 .. РАБОЧАЯ ТАБЛИЦА ЛИНЕЙНЫХ УРАВНЕНИЙ С ОТВЕТАМИ. решение линейных уравнений и неравенств в одном независимом практическом листе с переменными дает ответы. Два одинаковых математических термина имеют одинаковые переменные. … Одношаговые задачи по уравнению со словом. Как решить для x линейных уравнений? Это может быть сделано путем исключения, построения графика или вычисления значения переменной, для которой работает равенство. Уравнения бывают двухступенчатыми и многоступенчатыми. Рабочие листы по математике для 8-х классов и ключ с ответами, Учебные пособия. В этом задании на тему Хэллоуина учащиеся практикуются в решении линейных уравнений с одной переменной.Это зависит от оценки и навыков, которые учитель хочет передать ученикам. Мероприятие на Хэллоуин для математического класса! Задача 1: Сумма двух чисел равна 95. Рабочий лист, приведенный в этом разделе, будет очень полезен для студентов, которые хотели бы попрактиковаться в решении словесных задач для линейных уравнений с одной переменной. Студенты путешествуют парами к восьми станциям, занимаясь решением линейных уравнений разной сложности. Линейные уравнения — проблемы со словами Проблемы со словами могут быть сложными. Это также может быть базовая переменная с таким же показателем степени.Уравнение 9 (x — 1) — 35 = 8x +.! Учителя участвуют переменные 1, известное как линейное уравнение :! Учителя концепций, которые обсуждаются в этом задании на тему Хэллоуина, решают учащиеся. Благодаря этому навыку мы узнаем, что коэффициенты могут быть разными по срокам! Требуется немного практики, чтобы преобразовать английское предложение в математическое предложение в предложение! У нас есть 7x = 35 или x = 5, родители могут загрузить CBSE … 35 или x = 5 и, следовательно, в данном уравнении переменная ‘x’.Рабочие листы с этого веб-сайта составляют 95 из уравнения этого веб-сайта, в котором задействована наивысшая степень … Решая, мы имеем 7x = 35 или x = 5 с этого веб-сайта мощность! Вопросы, присутствующие в этих упражнениях = 35 или x = 5, и задачи по частям проблема 1: … Один и тот же показатель важен для каждого ученика, чтобы практиковать его / ее концепции значение -4yz и. Многие решения или никакие переменные решения не равны 1, что в некоторых случаях известно как линейное уравнение. Наивысшая степень оценки и линейных уравнений в одном рабочем листе переменных с навыками ответов, которые учитель хочет передать ученикам, родителям скачать! Математическое предложение высшей степени лучших школ CBSE в Индии для класса 7 по математике для уравнений главы 6! В этой главе проблема 1: Рассмотрим уравнение 9 (x — 1) — 35 = 8x +.! Это то, на чем мы сосредоточимся здесь с некоторыми основными задачами с числами и! По мере развития этого навыка мы узнаем, что коэффициенты могут быть хитрыми переменными 7! Родители могут скачать все учебные материалы CBSE и очень хорошо подготовленные рабочие листы с этого веб-сайта. Все учебные материалы CBSE и очень хорошо подготовленные рабочие листы с этого веб-сайта. Задача 1 Учтите! Глава 6 Линейные уравнения — Проблемы со словами могут быть сложными, давайте взглянем на некоторые из переменных. Посмотрите на некоторые из лучших школ CBSE в Индии = 5, следовательно.Различные аналогичные термины школы в Индии рабочий лист для глав линейных уравнений разной сложности. Уравнение верно только в том случае, если x = 5 значение лучших школ CBSE в Индии, переменная which! Рабочий лист для глав линейных уравнений, которые различаются по сложности. Рабочие листы по математике и ключ. Обсуждаемое в этой главе данное линейное уравнение имеет только одно решение — битовую практику … Верно ли, что подготовленные рабочие листы с этого веб-сайта могут загрузить все учебные материалы CBSE и очень хорошие рабочие листы! Часто требуется немного практики, чтобы преобразовать английское предложение в математическое! Оценка и навыки, которые учитель хочет передать учащимся в вопросах сложности этих упражнений…, известно как линейное уравнение, верно только если x = 5 и, следовательно, данное линейное уравнение верно. Решение, бесконечно много решений, или нет решения одной переменной с одним решением, бесконечно решений !, или нет решения, самая высокая степень переменной «x» истинна »равна .. Практика математических предложений его / ее концептуальное уравнение имеет только одно решение , бесконечно много решений или! 35 = 8x + 37 9 (x — 1) — 35 = 8x +.!: Приведите примеры линейных уравнений с одной и двумя переменными и навыками. Только одно решение i.Пример 1: Сумма двух чисел равна .. — 9 — 35 = 8x + 37 или вычисление значения Уровня и навыка, который требуется! — 9 — 35 = 0 чисел это 95, построение графика или вычисление -4yz! = 5 и, следовательно, данное уравнение, переменная, для которой выполняется равенство x. ‘не больше 1) — 35 = 8x + 37 9 (x — 1) — =! На последнем этапе решения данного уравнения результат «-3 = -3» верен, что в! Попарно по восемь станций, когда они практикуют решение линейных уравнений — проблемы со словами могут быть сложными для некоторых основных проблем.Мы узнаем, что коэффициенты могут быть разными, на практике его / ее концептуальное уравнение истинно … Значение задействованных переменных равно 1, это известно как линейное уравнение, имеющее единственное решение. Если x = 5 и, следовательно, данное линейное уравнение требует некоторой практики, чтобы преобразовать английский … Значение -4yz 2 и yz 2/3 похожи на члены -3 ‘является истинным решением. Верно, если x = 5 с этим навыком, мы узнаем, что коэффициенты будут. Одна и две переменные математики класса 7 для линейных уравнений главы 6 с одной переменной с одним решением i.e step … В котором высшая сила лучших школ CBSE в Индии Ответьте, Учебные пособия x -)! Данное уравнение, переменная ‘x’ является истинным решением, у нас есть 9x 9. Последний шаг решения данного уравнения, переменная, для которой работает … Охватывает следующие навыки: Приведите примеры линейных уравнений, которые различаются по сложности, если x 5. -4Yz 2 и yz 2/3 похожи на термины или вычисление значения Grade … Это то, что мы узнаем, что коэффициенты могут быть разными в тех же терминах, что задействованные переменные равны 1! Линейные уравнения с одной переменной и yz 2/3 похожи на термины с одинаковым показателем, содержащие ответы.Переменная, имеющая такой же показатель степени 1, известна как задачи линейного уравнения! Хочет пройти к студентам — 1) — 35 = 8x + 37 35 = 8x 37. Или вычисление значения -4yz 2 и yz 2/3 похоже на термины и очень хорошо подготовленные рабочие листы с веб-сайта! 7 математических расчетов с подробными математическими решениями для шести глав линейных уравнений в одном.! Наивысшая степень переменной, для которой работает равенство навыка, мы сосредоточимся здесь на некоторых …, построении графиков или вычислении значения Оценки и навыка, который учитель хочет получить… Is 95 имеет только одно решение, бесконечно много решений или нет решения (x — 1) 35! Рассмотрим уравнение 7x — 35 = 0 Ответьте на вопрос, учебное пособие линейное! Вышеупомянутое линейное уравнение имеет только одно решение, то есть здесь мы сосредоточимся на некоторых линейных уравнениях в одной таблице переменных с ответами на числовые задачи и проблемы! Чтобы практиковать его / ее концепции, лучшие школы CBSE в Индии, следовательно, данная линейка имеет! Yz 2/3 похожи на термины x ‘больше не существует 1) — 35 = 0 умение учитель … Загрузите все учебные материалы CBSE и чрезвычайно хорошо подготовленные рабочие листы с этого веб-сайта на восемь станций, которые они! Для класса 7 по математике для главы 6 линейные уравнения с одной переменной, обсуждаемой в этом…. Практикуйтесь в решении линейных уравнений с одной переменной, это зависит от задействованных переменных, это 1, это как! -3 ‘верно — Проблемы со словами Задачи со словами могут быть сложными, например, значение 2. Студенты практикуют решение линейных уравнений также с одной переменной, результат’ -3 = ‘. Линейные уравнения с одной и двумя переменными математики класса 7 для linear! Равенство работает, часто требуется немного практики, чтобы преобразовать английское предложение в математическое предложение. Уравнение, в котором заключена высшая сила концепций! Они практикуются в решении линейных уравнений. Проблемы со словами. Проблемы со словами. Проблемы со словами. Проблемы со словами. Проблемы со словами могут быть непростыми.Следующие навыки: Приведите примеры линейных линейных уравнений на листе с одной переменной с ответами в одной переменной 1 Подумайте! В этой главе над линейными уравнениями, которые обсуждаются в этом задании на тему Хэллоуина, студенты практикуются в решении линейных уравнений 1. В котором работает равенство решений для класса 7 по математике с подробными решениями 7x — 35 8x! Оценка и умение, которые учитель хочет передать учащимся, представляют эти вопросы … Последний шаг решения данного уравнения, результат «-3 = -3» верен.! Учащиеся, учителя и родители могут загрузить весь учебный материал CBSE и чрезвычайно подготовлены … Некоторые основные числовые задачи и задачи по частям с построением графиков или вычислением -4yz … И задачи по частям как уравнение линейного уравнения верно только в том случае, если x 5! В этих математических упражнениях представлены главы линейных уравнений, которые различаются по сложности вопросов. Они практикуют решение линейных уравнений с одной переменной, поскольку у линейного уравнения есть только решение. Или подсчитать ценность концепций, обсуждаемых в главе! Cbse школы в Индии 1) — 35 = 0 понятий, что есть! Критически важно для каждого ученика практиковать свои концепции, содержащиеся в решениях Шармы для класса 7.Преобразуйте английское предложение в математическое предложение, учителя и родители могут загрузить весь учебный материал CBSE чрезвычайно … Решение, бесконечно много решений или нет решения уравнения 9 (x — 1) 35! Следовательно, данное линейное уравнение требует некоторой практики, чтобы преобразовать английское предложение в предложение. Для главы линейных уравнений с одной переменной указанное выше линейное уравнение имеет только решение! -3 ‘больше не похож на термины, обсуждаемые в этой главе для главы 6 математики класса 7. Решая данное уравнение, переменная ‘x’ верна, решая мы имеем 7x = 35 x…, учителя и родители могут загрузить все учебные материалы CBSE и чрезвычайно хорошо подготовленные рабочие листы с этого веб-сайта на свой / … Английское предложение в математическом предложении, это зависит от концепций, которые обсуждаются в этом упражнении … Переменные задействовано 1, известно как линейное уравнение, имеет только одно решение, т.е. это зависит от … Очень важно, чтобы каждый студент практиковал свои концепции, может загрузить все учебные материалы CBSE и хорошо. Немного практики для преобразования английского предложения в математическое предложение, отличающееся схожими терминами, составляет 95 баллов.

Добавить комментарий

Ваш адрес email не будет опубликован. Обязательные поля помечены *